Vous êtes sur la page 1sur 103

Otorrinolaringología

Absceso mastoideo de otitis media aguda y cronica


Nivel de manejo del médico general: Diagnóstico Sospecha Tratamiento Inicial
Seguimiento Derivar

Aspectos esenciales

 Toda otitis de más de 2 semanas obliga a descartar la presencia de complicaciones


(una de las cuales es el absceso mastoideo).
 Su manejo constituye una urgencia y el paciente debe ser hospitalizado y recibir
antibiótico endovenoso.
 Si bien la clínica es orientadora, el TAC es el examen de elección para confirmar el
diagnóstico.
 El seguimiento es de resorte de especialista.

Caso clínico tipo

Paciente de 5 años con antecedentes de OMA a repetición, consulta en SU por otalgia


intensa del oído izquierdo asociado a aumento de volumen retroauicular con
desplazamiento del pabellón hacia anterior. A la otoscopía destaca un tímpano abombado y
de aspecto congestivo.

Definición

Compromiso infeccioso de las celdillas neumatizadas de la región mastoidea, asociado a


compromiso óseo (osteólisis), que se puede presentar como complicación de una OMA o
una OMCr.

Etiología-epidemiología-fisiopatología

En OMA los patógenos responsables son Streptococo pneumoniae (el más frecuente, 28-
58%), Haemophilus influenza, Moraxella catarrhalis, Streptococcus pyogenes. Y, en
OMCr, Bacilos Gram negativos y anaerobios, destacando Proteus mirabilis, Pseudomona
aeruginosa, Escherichia coli y Stafilococo aureus. Del total de complicaciones
intratemporales, en las OMA la Otomastoiditis es la más frecuente (sobre todo en niños, en
donde constituye una de las formas de presentación inicial de la OMA), en tanto que en las
OMCr es la segunda en orden de frecuencia (y es más habitual en adultos). Es en todos los
casos la complicación supurativa más frecuente. La formación del absceso implica la
diseminación por continuidad de los patógenos a las celdillas mastoídeas, con inflamación
de las mismas. A medida que avanza la infección el aumento de presión al interior de las
celdillas óseas por la inflamación, favorece la reabsorción de los tabiques óseos entre ellas,
formando una gran cavidad infectada retroauricular, con posibilidad de fistulizar hacia la
piel retroauricular o a la pared posterior del CAE. Este proceso suele preceder al resto de
complicaciones intratemporales (petrositis y laberintitis).

Diagnóstico

El diagnóstico es esencialmente clínico.

Anamnésis: Persistencia o evolución tórpida de otalgia, dolor retroauricular (en relación al


proceso mastoides), fiebre y compromiso del estado general.

Ex. Físico: Desplazamiento del pabellón auricular hacia anterior, signos de fluxión
retroauricular con aumento de volumen eritematoso y doloroso a la palpación, edematoso y
caliente. Puede haber fístula cutánea local o absceso cervical.

Los síntomas más frecuentes son el dolor, signos de fluxión retroauricular y desplazamiento
del pabellón auricular. Cerca de la mitad de los casos tienen el antecedente de una
complicación previa de una otitis media, y/o haber recibido curso de antibiótico reciente
por otitis media.

Otoscopía: puede encontrarse pared posterior del CAE con abombamiento hacia anterior,
sensible, con o sin otorrea. OMA: si se logra apreciar el tímpano, éste se encontrará
abombado y congestivo con o sin perforación y pus. OMCr: hallazgo de perforación
timpánica, con pus pulsátil desde caja timpánica, con o sin retracción del remanente
timpánico y presencia o no de colesteatoma.

Imágenes: De elección es el TAC, observándose velamiento de las celdillas mastoideas


asociado a destrucción de las mismas. Sólo se considera complicación si hay destrucción
ósea dado que el velamiento mastoideo se encuentra en cuadro de otitis medias no
complicadas (OMA u OME). Si se sospecha complicación intracraneana solicitar RM para
complementar estudio.

Tratamiento

 Hospitalizar
 Manejo sintomático; antiinflamatorio y antipiréticos.
 OMA: Cefalosporina 2º o 3º EV por 7 días, luego oral hasta completar 14 días.
 OMCr: Cefalosporina 3º (ceftazidima, cefotaxima) más antianaerobios
(clindamicina) EV. Duración del tratamiento: 14-21 días.
 Si a las 48 horas, no hay evolución favorable con tímpano íntegro o hay otras
complicaciones asociadas (ej: parálisis facial) puede hacerse miringotomía. La
mastoidectomía se reserva para aquellos que no tienen respuesta a manejo médico
con o sin miringotomía según el caso y para los que desde el diagnóstico presentan
otra complicación intratemporal o intracraneana.

Seguimiento
El seguimiento de estos pacientes es de resorte de especialista en Otorrinolaringología..

Absceso periamigdalino
Nivel de manejo del médico general: Diagnóstico Específico Tratamiento Completo
Seguimiento Completo

Aspectos esenciales

 Frecuente en adultos jóvenes.


 El agente etiológico más frecuente es Streptococo B hemolítico grupo A. (S.
Pyogenes)
 El diagnóstico es clínico.
 El tratamiento es drenaje quirúrgico asociado a terapia ATB parenteral.

Caso clínico tipo

Paciente de 21 años, consulta por odinofagia intensa y fiebre 38°C, asociada a sialorrea y
trismus de 48 horas de evolución. Recibió PNC benzatina en SAPU y hace 1 día está
usando amoxicilina 1 gramo cada 12 horas. Al examen físico amígdala derecha aumentada
de volumen, desplazada hacia medial, con edema del pilar anterior; ud. nota que el paciente
tiene voz de "papa caliente".

Definición

Infección aguda localizada entre la cápsula amigdalina por medial y el músculo constrictor
superior de la faringe por lateral.

Etiología-epidemiología-fisiopatología

Son raros en los niños, pero frecuentes en los adultos jóvenes, suelen verse en el curso de
una faringoamigdalitis aguda como complicación de la misma o ser un cuadro
independiente debido a una infección de las Glándulas de Weber.

El agente etiológico más común es el Streptococo pyogenes, seguido por el Streptococo


viridans, Staphylococus aureus y anaerobios como Bacteroides, Peptostreptococus y
Fusobacterium. Frecuentemente son polimicrobianos.

Diagnóstico

El diagnóstico es clínico.

Clínica: En el curso de una amigdalitis aguda, habitualmente en tratamiento, aparece dolor


intenso unilateral, con sialorrea por dificultad al deglutir y trismus por irritación del
constrictor. Fiebre de cuantía variable, compromiso del estado general, halitosis y voz
engolada/empastada ("de papa caliente"). Al examen físico el pilar anterior y paladar
blando se encuentran abombados con úvula desplazada a contralateral.

Imágenes: En general no se necesitan. Solicitarlas solo si hay duda en el diagnóstico, si se


sospecha extensión a otros espacios del cuello o si no hay adecuada respuesta a tratamiento.
La imagen de elección es el TAC de cuello con contraste.

Diagnóstico diferencial: Epiglotitis (progresa más rapidamente), absceso retrofaríngeo (en


general sin trismus, tienen cierta rigidez de cuello, con pocos hallazgos peri-amgidalinos),
flegmón periamigdalino (zona no fluctuante a la palpación, punción aspirativa con escaso
pus, TAC con pus sin pared que lo delimite).

Tratamiento

Hospitalizar al paciente (solo en casos seleccionados por especialista podría recibirse


manejo ambulatorio posterior a drenaje).

Primero puncionar y aspirar para verificar ubicación y confirmar diagnóstico (recordar


cercanía de carótida), luego hacer pequeña incisión en la intersección entre líneas trazadas
en pilar anterior y base de úvula, drenar y aspirar.

Tratamiento antibiótico ev hasta completar 48 horas afebril, con buena tolerancia oral y
disminución de parámetros inflamatorios. Penicilina sódica en dosis antianaerobias (PNC
sódica 4 a 5.000.000 UI c/6h). En alérgico a beta-lactámicos; Clindamicina (600mg c/8h
iv). Posteriormente, traslape a vía oral, con amoxicilina-ácido clavulánico o clindamicina
300 mg c/8 h por 14 a 21 días.

Es importante diferenciar del flegmón periamigdalino dado que éste último requiere
tratamiento antibiótico sin drenaje.

Seguimiento

Derivar si es un segundo episodio para amigdalectomía.

Absceso profundo del cuello


Nivel de manejo del médico general: Diagnóstico Sospecha Tratamiento Inicial
Seguimiento Derivar

Aspectos esenciales
 El absceso periamigdalino es el más frecuente.
 Debe sospecharse por la clínica y confirmar espacio involucrado con TC de cuello
con contraste.
 Se debe iniciar tratamiento antibiótico inmediato para evitar complicaciones graves
como la trombosis séptica de la vena yugular interna.

Caso clínico tipo

Paciente de 32 años consulta por cuadro de fiebre, odinofagia intensa y dolor a la


movilización del cuello. Al examen físico destaca trismus importante y abombamiento
faríngeo.

Definición

Colección purulenta ubicada en alguno de los espacios profundos del cuello


(submandibular, sublingual, masticatorio, parotídeo, visceral, periamigdalino, parafaríngeo,
retrofaríngeo, prevertebral y espacio peligroso).

Etiología-epidemiología-fisiopatología

Las principales causas de abscesos en espacios profundos del cuello son:

– Infecciones amigdalinas y faríngeas.

– Infecciones y abscesos dentarios.

– Procedimientos quirúrgicos orales.

Los agentes aislados con mayor frecuencia son estrepcocos (viridans y beta hemolítico),
estafilococos y anaerobios (Peptoestreptocco y Fusobacterium, entre otros). Habitualmente
las infecciones son polimicrobianas con aerobios en el 100% de los casos y anaerobios en
el 50%. La infección del espacio periamigdalino es el más frecuente a toda edad.

Diagnóstico

Dependiendo del espacio afectado puede haber trismus (parafaringeo), torticolis (pre-
vertebral), asimetría cervical (parafaringeo y retrofaríngeo), zonas fluctuantes,
desplazamiento de la pared faríngea con disfagia y dificultad respiratoria, déficit
neurológico (parafríngeo retroestiloideo, carotideo). El síntoma más constante es el dolor.
Signos más constantes son el aumento de volumen y fiebre. La aparición de estridor y
disnea son sugerentes de obstrucción de vía aérea ya sea por expasión local o diseminación
a mediastino. Confirmación diagnóstica con TC cervical con contraste de primera elección.

Complicaciones: Obstrucción de la vía aérea, Aspiración pus a vía aérea, Shock séptico,
Complicaciones vasculares (émbolos sépticos, trombosis yugular, estallido carotídeo),
Mediastinitis, Fasceitis necrotizante, Osteomielitis, etc.
Tratamiento

Tratamiento antibiótico con Penicilina sódica a dosis anaerobia (5.000.000 U cada 6 horas
ev) o ceftriaxona 1 g cada 12 horas asociado a clindamicina. Está contraindicado el drenaje
intraoral por eventual daño al paquete vascular del cuello (exceptuando drenaje de
retrofaríngeos). Si se decide, debe ser realizado por vía externa a través de cervicotomía por
el especialista. La duración del tratamiento es de 14 a 21 días, siendo endovenoso hasta 48
horas después que el paciente esté afebril.

Seguimiento

Al momento del diagnóstico, se debe iniciar terapia antibiótica y derivar a centro


secundario para eventual resolución quirúrgica.

Alergia nasosinusal
Nivel de manejo del médico general: Diagnóstico Específico Tratamiento Completo
Seguimiento Completo

Aspectos esenciales
 Patología muy frecuente, afectando aproximadamente al 25% de la población.
 Interfiere en calidad de vida.
 Diagnóstico no es de descarte (antecedentes, examen endonasal, eosinófilos nasales,
test cutáneo respiratorio).
 Puede coexistir con otras patologías rinosinusales.
 El tratamiento debe ser enfocado al síntoma principal.

Caso clínico tipo


Paciente de 13 años, consulta por rinorrea acuosa abundante, hiposmia y congestión nasal
de larga data. Antecedente de madre con rinitis alérgica. Al examen físico presenta mucosa
nasal pálida. Menciona que los síntomas se exacerban durante la primavera.

Definición
Inflamación de la mucosa nasal y con frecuencia de la mucosa del resto de las cavidades
perinasales, de causa alérgica.
Epidemiología
La rinitis alérgica constituye el 3% del total de las consultas médicas. Es una de las
condiciones crónicas más frecuentes en la población (18 a 25%). Puede traer un gran
deterioro en la calidad de vida en la población más activa.

Fisiopatología

Existe una reacción de hipersensibilidad tipo I, mediada por Ig E, con sensibilización


previa. Posee carga genética. Dependiendo de tipo de alérgeno podrá ser estacional
(pólenes, arboles) o perenne (ácaros del polvo, pelo de animales, ciertos alimentos).

Diagnóstico
Es fundamental indagar en la anamnesis por antecedentes de familiares: asma, rinitis
alérgica, dermatitis atópicas. Con un padre alérgico existe un 50% probabilidad y con los
dos padres 80%. Exacerbación estacional de síntomas, comorbilidades (IRA, sinusitis,
otitis, asma).

La clínica se expresa por: síntomas obstructivos (congestión nasal, hiposmia, rinorrea


acuosa y descarga posterior); síntomas pruriginosos (prurito nasal y conjuntival,
estornudos) y; síntomas irritativos (epistaxis y costras). Al examen físico los cornetes
muestran una palidez característica.

Los exámenes de laboratorio que resultan útiles son: test cutáneo a alérgenos respiratorios a
cualquier edad, pero mayor sensibilidad desde los 2 años; eosinófilos en secreción nasal (>
15%) y en sangre; e IgE específica.

Tratamiento
Se basa en 3 pilares:

1. Evitar contacto con alérgenos: Plumas, polvo, pasto. Usar filtros de aire, evitar
alfombras, pisos de fácil aseo, detergentes.
2. Farmacoterapia:

 Antihistamínicos: orales e intranasales, bloquen la histamina al ocupar su receptor.


Útiles en disminuir la secreción, el prurito nasal y ocular.
 Esteroides: intranasales o sistémicos, efecto antiinflamatorio. Disminuyen
estornudos, obstrucción nasal, rinorrea y edema.
 Descongestionantes: intranasales y sistémicos, generan vasoconstricción. Útiles en
obstrucción nasal. No se usan en niños.
 Cromoglicato de Sodio: Estabilizan los mastocitos, para que no liberen los
mediadores preformados, ni se neoformen.
1. Desensibilización: Disminuye la repuesta alérgica al contacto con el alérgeno, es
decir, la inmunoglobulina formada ataca al anticuerpo antes que el antígeno
desencadene la reacción. Se utiliza cuando ha fracasado el tratamiento
farmacológico y con test cutáneo positivo. Es el único tratamiento curativo.

Seguimiento
Debe ser realizado por médico general.

Alergia nasosinusal
Nivel de manejo del médico general: Diagnóstico Específico Tratamiento Completo
Seguimiento Completo

Aspectos esenciales
 Patología muy frecuente, afectando aproximadamente al 25% de la población.
 Interfiere en calidad de vida.
 Diagnóstico no es de descarte (antecedentes, examen endonasal, eosinófilos nasales,
test cutáneo respiratorio).
 Puede coexistir con otras patologías rinosinusales.
 El tratamiento debe ser enfocado al síntoma principal.

Caso clínico tipo


Paciente de 13 años, consulta por rinorrea acuosa abundante, hiposmia y congestión nasal
de larga data. Antecedente de madre con rinitis alérgica. Al examen físico presenta mucosa
nasal pálida. Menciona que los síntomas se exacerban durante la primavera.

Definición
Inflamación de la mucosa nasal y con frecuencia de la mucosa del resto de las cavidades
perinasales, de causa alérgica.

Epidemiología
La rinitis alérgica constituye el 3% del total de las consultas médicas. Es una de las
condiciones crónicas más frecuentes en la población (18 a 25%). Puede traer un gran
deterioro en la calidad de vida en la población más activa.

Fisiopatología
Existe una reacción de hipersensibilidad tipo I, mediada por Ig E, con sensibilización
previa. Posee carga genética. Dependiendo de tipo de alérgeno podrá ser estacional
(pólenes, arboles) o perenne (ácaros del polvo, pelo de animales, ciertos alimentos).

Diagnóstico
Es fundamental indagar en la anamnesis por antecedentes de familiares: asma, rinitis
alérgica, dermatitis atópicas. Con un padre alérgico existe un 50% probabilidad y con los
dos padres 80%. Exacerbación estacional de síntomas, comorbilidades (IRA, sinusitis,
otitis, asma).

La clínica se expresa por: síntomas obstructivos (congestión nasal, hiposmia, rinorrea


acuosa y descarga posterior); síntomas pruriginosos (prurito nasal y conjuntival,
estornudos) y; síntomas irritativos (epistaxis y costras). Al examen físico los cornetes
muestran una palidez característica.

Los exámenes de laboratorio que resultan útiles son: test cutáneo a alérgenos respiratorios a
cualquier edad, pero mayor sensibilidad desde los 2 años; eosinófilos en secreción nasal (>
15%) y en sangre; e IgE específica.

Tratamiento
Se basa en 3 pilares:

3. Evitar contacto con alérgenos: Plumas, polvo, pasto. Usar filtros de aire, evitar
alfombras, pisos de fácil aseo, detergentes.
4. Farmacoterapia:

 Antihistamínicos: orales e intranasales, bloquen la histamina al ocupar su receptor.


Útiles en disminuir la secreción, el prurito nasal y ocular.
 Esteroides: intranasales o sistémicos, efecto antiinflamatorio. Disminuyen
estornudos, obstrucción nasal, rinorrea y edema.
 Descongestionantes: intranasales y sistémicos, generan vasoconstricción. Útiles en
obstrucción nasal. No se usan en niños.
 Cromoglicato de Sodio: Estabilizan los mastocitos, para que no liberen los
mediadores preformados, ni se neoformen.

2. Desensibilización: Disminuye la repuesta alérgica al contacto con el alérgeno, es


decir, la inmunoglobulina formada ataca al anticuerpo antes que el antígeno
desencadene la reacción. Se utiliza cuando ha fracasado el tratamiento
farmacológico y con test cutáneo positivo. Es el único tratamiento curativo.

Seguimiento
Debe ser realizado por médico general.
Amigdalitis aguda
Nivel de manejo del médico general: Diagnóstico Especifico Tratamiento Completo
Seguimiento Completo

Aspectos esenciales

 Afecta principalmente a niños 3 a 8 años, con peak 5-6 años.


 Principal agente es viral.
 Diagnóstico clínico, idealmente con Test Pack o cultivo faríngeo.
 El tratamiento disminuye el periodo sintomático, evita estado portador y disminuye
complicaciones.
 Tratamiento más utilizado es PNC Benzatina.

Caso clínico tipo

Paciente de 5 años consulta por cuadro de 12 hrs. de evolución de fiebre hasta 39°C, CEG y
odinofagia intensa. Al examen físico aumento de volumen amigdalino con congestión,
edema y exudado pultáceo en criptas amigdalinas, además de adenopatías cervicales
sensibles.

Definición

Inflamación de amígdalas, con o sin exudado.

Etiología-epidemiología-fisiopatología

Afecta cualquier edad pero principalmente a niños entre 3 y 8 años. Su etiología es


principalmente viral, solo un porcentaje es bacteriana, y entre estos puede ser por
streptococo betahemolítico grupo A. Otros agentes: stafilococo, streptococo grupo B,
neisseria, treponema pallidum.

Diagnóstico

Generalmente clínico: inicio brusco de aprox. 12 horas de fiebre sobre 38,5º C,


decaimiento, odinofagia intensa, cefalea, otalgia ocasional, vómitos y dolor abdominal
frecuentes en niños. Al examen físico presenta halitosis, eritema y aumento de volumen de
amígdalas. Puede haber exudado purulento y/o petequias en paladar blando. Hay
adenopatías subangulomaxilares sensibles.

Criterios de Centor: Exudado amigdalino; Adenopatías cervicales anteriores sensibles;


Ausencia de tos; Historia de fiebre.
Cada factor: 1 punto (1 punto más entre 3 y 14 años, 0 punto entre 15 y 44 años, 1 punto
menos en mayor de 45 años).

3 ó más criterios positivos 40-60% VPP,

1 criterio 80% VPN. Sensibilidad y especificidad del 75%.

Lo ideal es siempre contar con apoyo de laboratorio: Cultivo faríngeo y/o Test de
diagnóstico rápido para SBGA (ELISA). Sensibilidad y especificidad del 90%.

 0-1 no tratar
 ≥2 test rápido + tratar
 4 tratar con antibióticos
 3-4 sin test tratar con antibióticos

Diagnóstico diferencial: faringoamigdalitis virales de comienzo súbito o con pródromos,


odinofagia más intensa y síntomas catarrales (Adenovirus, Coxsackie); Sd.
Mononucleosido, rinofaringitis.

Complicaciones:

 Generales: Fiebre reumática, glomerulonefritis post-estreptocócica.


 Locales: Adenitis supurativas, abscesos cervicales, abscesos periamigdalinos (la
más frecuente), abscesos parafaríngeos.

Tratamiento

Medidas generales: Reposo relativo, líquidos abundantes según tolerancia.

Medicamentos: manejo sintomático. Paracetamol: 15 mg/kg/dosis, cada 8 horas en niños,


500 mg c/8 h en adulto. Ibuprofeno 10 mg/kg c/8h en niños, y 400-600 mg c/8h en adultos.
Antibióticos: Penicilina Benzatina contraindicada en < de 4 años; < 25 kilos: 600.000 U IM
por 1 vez; > 25 kilos: 1.200.000 U IM por 1 vez.

Alternativa: Amoxicilina 75 mg/kg./día cada 8 ó 12 horas por 10 días. En caso de alergia a


PNC: claritromicina 15 mg/kg./día dividido en 2 dosis, por 10 días. Azitromicina 12
mg/kg./dosis única durante 5 días. Cefadroxilo (1° generación) por 10 días Cefuroximo (2°
generación) por 10 días.

Seguimiento

Por médico general, derivar a ORL si amigdalitis a repetición: presencia de más de tres
episodios de amigdalitis estreptocócica en 6 meses o más de cinco en un año.
(Documentadas con clínica y/o laboratorio).
Amigdalitis crónica
Nivel de manejo del médico general: Diagnóstico Específico Tratamiento Inicial
Seguimiento Derivar

Aspectos esenciales

 Es un concepto CLÍNICO.
 El tratamiento definitivo es quirúrgico.
 Siempre usar analgesia

Caso clínico tipo

Varón de 7 años de edad, con historia de 6 cuadros de faringoamigdalitis en los últimos dos
años, tratadas siempre con antibióticos, consulta por cuadro de 3 días de evolución de
odinofagia, sensación febril y decaimiento. La madre refiere que su hijo “falta mucho al
colegio por la frecuencia de los cuadros, y se atrasa con las tareas debido a esto.”

Definición

Se define por la aparición recurrente de cuadros agudos inflamatorios amigdalinos, de


manera que produce trastornos en la convivencia familiar o en la escolaridad.

Etiología - epidemiología - fisiopatología

La etiología puede corresponder a una complicación de una amigdalitis aguda. La mayoría


de los cuadros afectan a niños y jóvenes. Rara vez afecta sobre los 50 años. La patología
puede corresponder a microabscesos delimitados por tejido fibroso en los folículos
linfoides de las amígdalas. La infección crónica de los senos paranasales o patología
infecciosa dental puede ser un factor predisponente.

Diagnóstico

El diagnóstico generalmente es clínico, según criterios diagnósticos de amigdalitis aguda.


Según la Academia Americana de ORL se considera para el diagnóstico de amigdalitis
crónica (o amigdalitis aguda recurrente) lo criterios de Paradise:

 7 episodios de amigdalitis en un año


 5 episodios de amigdalitis por año en 2 años consecutivos
 3 episodios de amigdalitis por año durante 3 años consecutivos.

El cuadro clínico debe tener:


 Fiebre mayor a 38ºC
 Exudado amigdalino
 Cultivo o test rápido positivo para estreptococo beta hemolítico A o en su defecto
test rápido positivo.

Tratamiento

Cada cuadro agudo debe tratarse según los esquemas terapéuticos señalados para
amigdalitis aguda. La amigdalitis crónica es una de las indicaciones de amigdalectomía.
Hay que reconocer sin embargo que esta no es una regla de oro para la indicación
quirúrgica y que en esta decisión debe considerarse también otros puntos: comorbilidad del
menor que pueda complicar la evolución, alteraciones en la calidad de vida, abuso de
terapia antibiótica mal indicada, asociación con obstrucción de la vía aérea superior,
aprehensiones y deseo de los padres, etc.

Seguimiento

Por especialista

Atresia coanal congénita


Nivel de manejo del médico general: Diagnóstico Específico Tratamiento Inicial
Seguimiento Derivar

Aspectos esenciales

Es una urgencia médica (bilaterales).

Alta asociación con otras alteraciones congénitas; realizar examen físico completo.

El diagnóstico es por visualización directa de la obstrucción con nasofibroscopía y se


complementa con TAC.

El tratamiento definitivo es quirúrgico.

Caso clínico tipo

Recién nacido, de 39 semanas de gestación, de peso y talla adecuados al nacimiento,


comenzó con cuadro insidioso de dificultad respiratoria, presentando cianosis importante,
necesitando maniobras de resucitación, debiendo ser intubado posteriormente. Al examen
físico se pesquisa una obstrucción bilateral de las cavidades nasales, por ocupación de
tejido.

Definición

Bloqueo de la vía aérea nasal a nivel de las coanas (comunicación de las fosas nasales con
la rinofaringe), debido a ocupación por tejido óseo, osteomembranoso o, raramente, sólo
membranoso. Puede ser uni o bilateral (hasta en la mitad de los casos).

Etiología-epidemiología-fisiopatología

Su etiología es desconocida. Se piensa que se debe a la persistencia postnatal de una fina


capa de tejido que separa la nariz de la boca en el período fetal (membrana bucofaríngea).
La ACC es la anormalidad nasal congénita más común, y es dos veces más frecuente en
mujeres. Más del 50% de los RN afectados presenta otras afecciones congénitas asociadas.

Diagnóstico

Se sospecha en la atención inmediata del RN al no poder introducir la sonda nasal.

Bilateral: Se sospecha en RN que presentan dificultad respiratoria persistente, que


disminuye cuando lloran (en donde respiran por la boca) y dificulta la alimentación. Los
bebés pueden presentar cianosis cíclica, e incluso pueden llegar a requerir resucitación al
momento de nacer.

Unilateral: La sospecha y el diagnóstico son más tardíos. El paciente presenta obstrucción


nasal y rinorrea unilateral. Al examen físico se observa la obstrucción con nasofibroscopía
flexible. Se puede realizar TAC de cavidades que mostrará los componentes óseo y
membranoso.

Tratamiento

Lo más importante es resucitar al bebé de ser necesario, pudiendo llegar a requerir


intubación o traqueostomía. El tratamiento definitivo es la cirugía, que puede realizarse por
vía transnasal o traspalatina, que puede no ser inmediata si es que el bebé aprende a respirar
por la boca. En los casos unilaterales se puede diferir según la prioridad de manejo que
requieran las malformaciones asociadas.

Seguimiento

Por especialista.
Barotrauma ótico y trauma acústico agudo
Nivel de manejo del médico general: Diagnóstico Específico Tratamiento Inicial
Seguimiento Derivar

Aspectos esenciales

Exposición a ruido de gran intensidad, con aparición inmediata de síntomas.

Diagnóstico clínico + audiometría.

Siempre sospechar fístula peri y/o endolinfática.

Barotrauma: Facilitado por disfunción tubaria previa, ante cambios bruscos de presión
ambiental.

Prevención es esencial.

Caso clínico tipo

Varón de 25 años consulta por hipoacusia moderada del oído derecho de 2 días de
evolución, luego de que por accidente disparara un arma de caza dentro de su pieza. En el
examen audiométrico se observa una brusca caída de la audición en frecuencias alrededor
de los 4Khz.

Definición

Trauma acústico agudo: Hipoacusia producida por exposición a un ruido intenso de corta
duración. Puede ser uni o bilateral. Barotrauma ótico: Trauma provocado por diferencia de
presión.

Etiología-epidemiología-fisiopatología

La exposición a estímulos sonoros excesivos (>135 dB), lleva a una fatiga auditiva. Esto
implica una modificación transitoria del umbral de audición por la vasocontricción que
produce el ruido en la microcirculación de la cóclea. Esta pérdida transitoria del umbral se
recupera a las 16 horas después del cese del ruido. Estas exposiciones intensas y cortas
llevan a un daño mecánico del órgano de Corti irrecuperable, con lesión a nivel de las
células ciliadas externas cerca de ventana oval (que sensan frecuencias alrededor de los
4.000 Hz). Si el ruido es de gran intensidad, puede llevar a la rotura de la membrana
timpánica y, en ocasiones a rotura de la membrana laberíntica con fístula peri y/o
endolinfática. El barotrauma se observa en los vuelos en avión y en las prácticas de
deportes submarinos y se explica por la incapacidad de la trompa de Eustaquio de igualar la
presión. El problema se ve facilitado cuando existe previamente una disfunción tubaria. Los
cambios de presión bruscos pueden llevar a una destrucción de las membranas del oído
interno, dando lugar a una hipoacusia sensorioneural y/o vértigo. También puede
producirse una fístula perilinfática

Diagnóstico

En el trauma acústico existe el antecedente de exposición a un estímulo sonoro de gran


intensidad, seguido de síntomas como hipoacusia y tinnitus. En todo trauma acústico agudo
se debe sospechar una fístula de perilinfática hacia el oído medio, principalmente si existe
anacusia. Se debe buscar también rotura de membrana timpánica a la otoscopia. A la
audiometría es característico el escotoma auditivo, una caída específica de la audición en
frecuencias entre 4.000-6.000 Hz, volviendo a la normalidad a los 8.000 Hz. En el
barotrauma, el antecedente de exposición a cambios bruscos de presión (vuelos en avión,
buceo) es fundamental. Como síntoma presenta sensación de presión ótica, hipoacusia y
otalgia uni o bilateral. Al examen con otoscopio la membrana puede estar retraída,
hiperémica o hemorrágica, y puede existir exudado seroso intratimpánico e incluso
hemotímpano.

Tratamiento

En el trauma acústico, una vez producido el daño del órgano de Corti no existe tratamiento,
por lo que la prevención es indispensable. En caso de presentarse fístula perilinfática se
debe explorar quirúrgicamente el oído medio de inmediato, para buscarla y repararla,
tratando de salvar la audición. En caso de rotura timpánica se puede esperar su resolución
espontánea si es pequeña (2-6 meses), o su reparación quirúrgica si es de mayor tamaño o
no repara espontáneamente. En el caso del barotrauma, se pueden utilizar
descongestionantes y antihistamínicos antes de andar en avión o bucear a modo de
prevención. Una vez ocurrido el daño los descongestionantes y antihistamínicos son poco
efectivos. Está indicado el uso de antibióticos cuando hay ruptura timpánica y se ha
contaminado el oido medio. Es una situación de emergencia cuando el trauma rompe el
oido interno y se forma una fístula perilinfática. Esto debe sospecharse cuando el paciente
presenta vértigo e hipoacusia sensorioneural. Esto requiere derivación inmediata. La
prevención del cuadro es fundamental y así se deben impartir instrucciones en el sentido de
evitar volar y practicar buceo a aquellos con problemas que afecten el funcionamiento de la
tuba auditiva. Se recomienda además en los vuelos las maniobras que abren la tuba
(masticar, deglutir, bostezar), o bien maniobras de Valsalva.

Seguimiento

Por especialista.

Cáncer de boca, labios y orofaringe


Nivel de manejo del médico general:
Diagnóstico Sospecha Tratamiento Inicial Seguimiento Derivar.

Aspectos esenciales

 Histología más común: cáncer epidermoide.


 Más frecuente en hombres, 5º y 6º década.
 Factores de Riesgo más importantes: tabaco y alcohol.
 Principal Factor Pronóstico: compromiso linfonodal.
 Diagnóstico: clínica más confirmación histológica por biopsia.
 Etapificación: TNM.
 Tratamiento: combinación de cirugía, quimioterapia y radioterapia.

Caso clínico tipo

Paciente sexo masculino 65 años, fumador de IPA 40, que refiere hace 3 meses
aparición de adenopatía cervical unilateral. Al examen físico destaca lesión
blanquecina en cara lateral de la lengua, indolora, con tendencia a la ulceración y
sangrado.

Definición

Neoplasia maligna originada en labios, cavidad oral (desde unión de labios, hasta
la unión del paladar duro y blando por superior, y papilas circunvalares por
inferior) y orofarínge (desde paladar blando hasta epiglotis).

Etiología - epidemiología - fisiopatología

El Cáncer intra-oral es la 6ta causa de cáncer en el mundo. Corresponde al 30% de


los cánceres de cabeza y cuello. Es más frecuente en hombres en la 5º y 6º
década de la vida. La histología más común es el carcinoma epidermoide (90%),
pudiendo encontrar también carcinoma basocelular a nivel del labio superior,
y adenocarcinomas de glándulas salivales menores a nivel del paladar. La
ubicación más frecuente es en lengua móvil.

Factores de riesgo:

 Abuso de alcohol y tabaco (mayor riesgo si están combinados),


 Irritación crónica (dentaduras enfermas, mala higiene oral, prótesis
inadecuadas, radiación actínica (labio),
 Infección por Virus papiloma.
 Presencia de lesiones premalignas como: leucoplasia, eritroplasia,
hiperplasia y displasia.
 La diseminación es principalmente por vía linfática a los linfonodos
cervicales.
Diagnóstico

Se inicia con la sospecha clínica seguida de un examen físico que debe incluir:
Palpación intra oral, otoscopía y examen de las zonas ganglionales. El estudio se
debe continuar con: Examen Endoscópico con toma de biopsia y búsqueda de un
tumor sincrónico (esófago y bronquios). Examen Radiológico (TC y/o RM), para
apreciar la extensión y su diseminación ganglionar, y Examen Dental de rigor.

La clínica varía según la localización. En todo tumor de cabeza y cuello es posible


encontrar adenopatías cervicales aisladas. El 40% de ellas son asintomáticas. En
labios se presenta como lesiones nodulares irregulares, con tendencia a la
ulceración y sangrado. En cavidad oral y orofaringe se puede presentar como
ulceración infiltrante, indolora y persistente, tumefacción, dificultad para mover la
lengua, otalgia refleja, movilidad dentaria, gingivorragia, odinofagia, anestesia de
V3 y disfagia.

Tratamiento

El tratamiento se decide en conjunto con un equipo multidisciplinario. Incluye


resección quirúrgica oncológica del tumor primario, disección linfática cervical,
según la etapificación, radioterapia complementaria, y reconstrucción lo más
estética y funcional posible.

Seguimiento

Derivar. Vigilancia estricta hasta los dos primeros años, luego anual hasta los 5
años. Después se consideran curados pero se mantienen en control. El 90% de las
recidivas aparecerá en los 2 primeros años. La sobrevida global es de 50% a 5
años.

Cáncer nasosinusal
Nivel de manejo del médico
general: Diagnóstico Sospecha Tratamiento Inicial Seguimiento Derivar

Aspectos esenciales

 La rinorrea unilateral purulenta en adultos debe hacer sospechar cáncer


nasosinusal.
 El carcinoma epidermoide es el más frecuente.
 La localización más frecuente es el seno maxilar.
 El tratamiento es quirúgico. Puede requerir de QT o RT.

Caso clínico tipo

Un paciente de 63 años, DM e HTA, consulta por presentar rinorrea purulenta por


fosa nasal derecha de larga data, tratada en varias ocasiones en APS sin
resultados. Se realizó Rx de CPN donde se observa masa en seno maxilar derecho

Definición

Neoplasia maligna en las cavidades nasales y/o paranasales.

Etiología - epidemiología - fisiopatología

Son poco frecuentes: 0.2% de todos los cánceres y 1.5% a 2% de los todos los
tumores de vía aérea superior. Afectan más a hombres adultos. Entre los factores
etiológicos, no se ha visto relación con el tabaco o el alcohol; parece jugar un rol
importante la exposición al polvo de madera (factor de riesgo de adenocarcinoma
de etmoides) y también a otros tóxicos como el níquel (factor de riesgo de
carcinoma epidermoide); el papiloma invertido es un precursor de carcinoma
epidermoide. El tipo histológico más frecuente es el epidermoide (70%), le sigue el
adenocarcinoma (20%) y del carcinoma indiferenciado. Otros son el
neuroestesioblastoma (tumor neuroendocrino del bulbo olfatorio), melanomas y
sarcomas. El tumor benigno frecuente de los senos paranasales es el osteoma.

Diagnóstico

El diagnóstico se realiza con la clínica y se completa con nasofibroscopía. Se


efectúan pruebas de imagen (TC) para buscar signos de destrucción ósea.
Respecto a la clínica pueden presentar obstrucción nasal, rinorrea unilateral
purulenta y epistaxis. Otras manifestaciones son las algias faciales. y
abombamiento maxilar con manifestaciones intraorales en algunos casos. Los
síntomas dependen en gran medida de la localización del tumor. Su localización
por orden de frecuencia es la siguiente: seno maxilar (50%), etmoides (30%) y
fosas nasales (20%). Los frontales y esfenoidales son excepcionales. No son
frecuentes las metástasis regionales (cervicales) ni a distancia.

Tratamiento

La cirugía es la opción más empleada, es la única opción curativa, asociada o no a


radioterapia; la quimioterapia suele utilizarse solo con fines paliativos. La
radioterapia se usa unida a la cirugía o en tumores radiosensibles (por ejemplo,
neuroestesioblastoma). La presencia de metástasis cervicales es un factor de muy
mal pronóstico.

Seguimiento

Por especialista. Frente a la sospecha de un paciente con obstrucción nasal,


rinorrea purulenta unilateral y/o epistaxis que no responde a tratamiento, debe ser
derivado a ORL para su estudio y manejo.

Cuerpo extraño en el esófago


Nivel de manejo del médico general: Diagnóstico Específico Tratamiento Inicial
Seguimiento Derivar

Aspectos esenciales

Urgencia frecuente.

Síntomas frecuentes; disfagia y sialorrea.

Generalmente existe el antecedente.

En niños frecuentemente monedas.

Extracción mediante esofagoscopia rígida.

Caso clínico tipo

Paciente de 5 años, sin antecedentes mórbidos de importancia, consulta ya que tras la


ingesta de una moneda presenta disfagia a sólidos y líquidos, sialorrea, sensación de CE y
disnea sin estridor. Se realizó radiografía simple de tórax y cervical que evidencio CE en el
tercio proximal del esófago. Se realizó esofagoscopia rígida extrayendo CE sin
complicaciones posteriores.

Definición

Es la presencia de un cuerpo extraño (CE) en el esófago. Es más frecuente en el tercio


superior de este, lo que se relaciona con la potencia de los músculos constrictores faríngeos,
la debilidad de la musculatura peristáltica y la presencia del primer estrechamiento
esofágico.
Etiología-epidemiología-fisiopatología

Existen 3 áreas de estrechamiento fisiológico en el esófago en las que puede alojarse un


cuerpo extraño:

- Esfínter esofágico superior (músculo cricofaríngeo).

- Nivel del arco aórtico.

- Esfínter esofágico inferior.

Es un motivo de consulta frecuente. La ingesta habitualmente es accidental en adultos,


excepto, en pacientes con demencia, déficit psíquico y niños. En niños lo más frecuente son
las monedas, en adultos depende de los hábitos alimenticios (huesos de pollo, espinas de
pescado, prótesis dentales).

Diagnóstico

Clínico, generalmente se entrega el antecedente. Con frecuencia es asintomático. Si


presenta síntomas, puede tener disfagia, odinofagia, sialorrea, dolor y disnea, que a menudo
son transitorias. En la exploración física lo primero que se debe examinar es la vía aérea y
respiración. Edema, eritema o crépitos en cuello sugieren perforación esofágica lo cual es
una emergencia quirúrgica. Si presenta estridor inspiratorio o sibilancias es sugerente de
cuerpo extraño esofágico con compresión traqueal. Para confirmar se recomienda una
radiografía simple que incluya cuello, tórax y abdomen en proyección anteroposterior y
lateral. Se debe buscar dirigidamente en las zonas de estrechamiento fisiológico del
esófago.

Tratamiento

Lo más común es que no requiera intervención. 10-20% de los casos requieren resolución
endoscópica. Menos del 1% termina en cirugía. Se requiere intervención urgente si presenta
algunas de las siguiente manifestaciones clínicas:

- Si el objeto es filoso, grande (mayor a 5 cm) y se encuentra en esófago o estómago.

- Si es un imán.

- Cuando es una batería y se ecuentra en esófago.

- Si presenta signos de compromiso de vía aérea.

- Cuando se evidencia que presenta una obstrucción completa del esófago (Ej. que no pueda
tragar).

- Si presenta signos de inflamación u obstrucción intestinal.


Manejo expectante. Si no se cumple con alguna de las características descritas se puede
tener un manejo expectante por 24 horas porque el paso espontáneo es común. La
intervención debe ser realizado por especialista mediante la extracción con esofagoscopia
rígida (generalmente bajo anestesia general), también se puede extraer mediante endoscopía
flexible con anestesia local.

Seguimiento

Debe ser realizado por el especialista.

Cuerpo extraño en el oído


Nivel de manejo del médico general: Diagnóstico Específico Tratamiento Inicial
Seguimiento Derivar

Aspectos esenciales

Principalmente en niños.

Conducta dependerá del objeto en cuestión, sintomatología del paciente y disponibilidad de


instrumental adecuado.

Pilas botón son motivo de derivación de urgencia a especialista.

Seguimiento por especialista.

Caso clínico tipo

Escolar de 7 años, traído por madre por irritabilidad y otalgia de horas de evolución. A la
otoscopía se visualiza fragmento de goma de borrar escolar. Se deriva a especialista para su
extracción.

Definición

Cualquier objeto que accidental o intencionalmente se introduce y aloja en el conducto


auditivo externo.

Etiología-epidemiología-fisiopatología

Se da principalmente en niños. Ocurre de manera accidental en adultos o asociados a


enfermedad psiquiátrica. Pueden ser objetos inanimados inertes (juguetes, trozos de papel,
goma, lápices, mostacillas, botones) o Inanimados no inertes (semillas, alimentos), o
químicos (pilas) o Animados (insectos). Tanto semillas como insectos vivos significan un
potencial daño a las paredes del CAE y del tímpano. En el caso de los insectos vivos, se
deben ahogar en aceite o vaselina líquida previo a su extracción por un especialista. Por
otro lado, las semillas no deben irrigarse puesto que su reacción a la hidratación es
dilatarse, lo cual dificultaría un intento de extracción, aumentando también el riesgo de
daño a la anatomía del oído, y empeorando la sintomatología. Es de especial importancia la
presencia de pilas botón, constituyendo una causa de derivación de urgencia. El daño se
produce por la liberación de sustancias químicas (propiciado por el ambiente húmedo del
CAE) alcalinas muy fuertes, capaces de generar daño severo a las pocas horas. El nivel de
daño dependerá de: la ubicación (profundidad en el CAE), el tiempo de exposición y la
carga previa de la pila. En este caso no intentar extracción. Derivar de urgencia a
especialista.

Diagnóstico

Según clínica: Antecedente de introducción casual o intencional de un cuerpo extraño.


Pueden pasar inadvertidos si son inertes. Puede presentar hipoacusia de conducción,
prurito, otalgia, otorrea, o sensación de movimiento.

Por visualización a la otoscopia

Derivar a especialista:

Pilas y químicos

Sin instrumental adecuado disponible

Objetos profundos

Sospecha de daño timpánico o al CAE

Insectos

Para la extracción:

Instrumental adecuado disponible: iluminación, aspiración, pinza.

Objeto visible, poca profundidad.

Inanimados inertes y semillas. NUNCA mojar semillas, aumentan de volumen.

Posterior a la extracción, se puede complementar con antibióticos tópicos (en gotas).


Otra alternativa para intentar extracción es realizar lavados de oídos (no en caso de
semillas). Consideraciones especiales: - Nunca lavar semillas. - Instilar aceite previo a
lavado en insectos. - Derivar rápidamente las pilas.

Seguimiento

Por especialista.

Cuerpo extraño en la vía aérea


Nivel de manejo del médico genral: Diagnóstico Específico Tratamiento
Inicial Seguimiento Derivar

Aspectos esenciales

Patología frecuente en ámbito pediátrico. 55%, niños de 6 meses a 4 años.

Diagnóstico por sintomatología, imagenología o examen físico.

Tratamiento inicial basado en estabilización de vía aérea y condición cardiorrespiratoria.

Caso clínico tipo

Preescolar, traído a SU. Se encontraba almorzando cuando inicia cuadro disnea de inicio
súbito asociado a "tos de perro". Al examen físico destaca cianosis perioral, estridor
inspiratoio, sin fiebre.

Definición

Presencia de un objeto en el trayecto de la vía aérea, pudiendo o no generar una obstrucción


que comprometa la ventilación del paciente y, eventualmente, la vida del mismo.

Etiología-epidemiología-fisiopatología

Principal causa de muerte otorrinolaringológica.

Grupo etáreo más afectado: niños entre 6 meses y 4 años.

Factores predisponentes en niños:

Fase oral del desarrollo, Risa, llanto, correr mientras comen.


Inmadurez neuromuscular (ineficacia esfínter laríngeo)

Falta de molares: no trituran bien la comida.

Condiciones anatomo-funcionales de la vía aérea de un niño:

Vía aérea estrecha y colapsable (tejido inmaduro)

Epiglotis alta en forma de U

Respiración nasal obligada en lactantes

Punto de mayor resistencia en cartílago cricoides

En pacientes de más edad se desencadena con mayor eficacia el reflejo de la tos, por lo que
la aspiración de un cuerpo extraño en ellos debe hacer sospechar una enfermedad
neuromuscular o efectos de alcohol y/o drogas.

Origen: 60% alimenticio; 40% otros.

Dentro de los alimentos:

80% de origen orgánico (maní, nueces, porotos, maíz, huesos, semillas): estos producen
una rápida reacción inflamatoria local.

20% de origen inorgánico (más frecuente en edad escolar): menor reacción inflamatoria,
dan menos sintomatología.

Ubicación:

-Bronquio derecho: Más frecuente ya que tiene menos angulación respecto a la tráquea que
el izquierdo.

-Bronquio izquierdo.

-Laringe

Diagnóstico[editar | editar código]

Anamnesis: Detallar inicio del cuadro, posibles causas de mismo, antecedentes médicos,
alergias, etc.

Clínica: se describen 3 fases:

Inmediata: tos, asfixia, cianosis, estridor, sibilancias.


Asintomática: de minutos a meses, según localización, tamaño del objeto, grado de
obstrucción, e inflamación generada.

Reactiva: producto de la infección del cuerpo extraño de larga data, dada por tos crónica,
expectoración, fiebre, sibilancias, hemoptisis ocasional, neumonías a repetición.

Detalle Según localización:

Bronquial (80%): más frecuente en bronquio principal derecho.

Fase de penetración: disfonía, estridor, tos (similar a anteriores).

Fase de fijación: Tos, disnea de intensidad variable.

Fase de complicaciones: dependerá del tipo de cuerpo extraño:

Válvula de libre paso (pseudoasma): sibilancias

Válvula de paso en un solo sentido: enfisema localizado

Cierre total: atelectasia

Otras: bronconeumonía, empiema, sepsism absceso pulmonar.

Laríngea (2-12%): principalmente en menores de 1 año.

Síndrome de penetración (crisis de asfixia por espasmo de la glotis)

Obstrucción completa: apremio respiratorio, cianosis, paro cardiorespiratorio y muerte.

Obstrucción parcial: estridor, disfonía, tos, odinofagia y disnea.

Traqueal (7%): tos, estridor, choque audible y palpable en inspiración del cuerpo extraño
con la glotis. No presenta disfonía.

Imagenología: Radiografía posteroanterior y lateral de cuello y tórax (objetos radiopacos)

Visualización directa en la radiografía

Signos indirectos: enfisema, atelectasia.

Los signos radiológicos de sospecha de aspiración de cuerpo extraño son el atrapamiento


aéreo y la atelectasia. Con sospecha de CE de localización bronquial se recomienda
solicitar radiografía de tórax posteroanterior en inspiración y espiración para evidenciar
atrapamiento de aire (hallazgo precoz).
Broncoscopía

TAC: mayor resolución, muestra objetos radiolúcidos, alto costo, se usa casi únicamente en
estudio de cuerpos extraños de larga data.

Diagnóstico de Certeza:

Estridores y sibilancias localizadas

Palpación cervical del cuerpo extraño

Hallazgos radiológicos compatibles

Tratamiento

Identificar naturaleza y sitio de obstrucción. Control de vía aérea bajo nivel de obstrucción.
Síndrome de penetración: En <1 año maniobra de Heimlich modificada, en mayores,
maniobra de Heimlich (solo si es obstrucción total, en obstrucción parcial está
contraindicado) En caso de no responder a la primera maniobra, identificar sitio de
obstrucción y establecer vía aérea bajo tal nivel:

Cricotiroidectomia de urgencia.

Traqueostomía (en pabellón)

Traqueostomia percutánea (en pabellón)

En pacientes inconscientes o en paro cardiorespiratorio: reanimación cardiopulmonar,


hospitalizar y establecer vía venosa. Derivar: extracción del cuerpo extraño en pabellón por
especialista, con anestesia general y broncoscopía rígida.

Seguimiento

Por especialista

Cuerpo extraño nasal


Nivel de manejo del médico general: Diagnóstico Específico Tratamiento
Inicial Seguimiento Derivar

Aspectos esenciales
Patología se presenta principalmente en población pediátrica.

Conducta dependerá del objeto en cuestión, su ubicación, y la disponibilidad de


instrumental adecuado.

Pilas botón son motivo de derivación de urgencia a especialista.

Caso clínico tipo

Preescolar traído por madre quien refiere cuadro de rinorrea unilateral de 3 semanas que
evoluciona con epistaxis ocasionales y mal olor. A la inspección se observa cuerpo extraño
impactado en mucosa nasal.

Definición

Cualquier objeto que accidental o intencionalmente se introduce y aloja en la cavidad nasal.

Etiología-epidemiología-fisiopatología

Se da principalmente en niños. Ocurre de manera accidental en adultos asociados o no a


enfermedad psiquiátrica.

Los objetos más frecuentes suelen ser: Papel, Plumavit, Plasticina, Juguetes, Monedas,
Pilas y alimentos (semillas, dulces).

Las semillas suelen dilatarse debido a la alta humedad presente en la cavidad nasal, lo cual
dificulta su extracción, aumenta el riesgo de daño a la anatomía de la cavidad, y empeora la
sintomatología del paciente.

Es de especial importancia la presencia de pilas botón, constituyendo una causa de


derivación de urgencia. El daño se produce por la liberación de sustancias químicas
(propiciado por el ambiente húmedo de la cavidad nasal) alcalinas y muy fuertes, capaces
de generar daño severo a las pocas horas: quemaduras intranasales graves y perforación
septal.

Si el cuerpo extraño se mantiene por mucho tiempo se puede formar un rinolito.

Diagnóstico

Anamnesis:

Antecedentes de introducción casual intencional o ccidental de un cuerpo extraño.

Rinorrea de mal olor, unilateral, incluso purulenta

Obstrucción nasal unilateral, hemorragia nasal


Alteración del olfato,

Sinusitis refractarias a tratamiento o recurrentes.

Examen físico:

Visualización directa del cuerpo extraño con rinoscopía

Epistaxis

Descarga posterior

Tratamiento

Para la extracción:

Instrumental adecuado disponible: iluminación, pinza, anestésico local y vasoconstrictor.

Sólo proceder si se puede retirar con pinzas desde vestíbulo nasal. El resto debe ser derivad
a centro especializado.

Posterior a la extracción, se puede complementar con antibióticos tópicos.

Derivar a especialista:

Pilas y químicos

Sin instrumental adecuado disponible

Objetos profundos

Riesgo de aspiración

Seguimiento

Por especialista.

Diagnóstico diferencial de hipoacusias de transmisión


Nivel de manejo del médico
general:Diagnóstico: Sospecha Tratamiento: Inicial Seguimiento: Derivar
Aspectos esenciales

 La hipoacusia de transmisión refleja un daño del oído externo o medio.


 Son fundamentales en su evaluación la audiometría e impedanciometría.
 La otoesclerosis se caracteriza por hipoacusia de transmisión bilateral progresiva.
 El tratamiento de la otoesclerosis es quirúrgico.

Caso clínico tipo

Una paciente de 35 años, con antecedentes familiares de hipoacusia, acude por presentar
hipoacusia progresiva bilateral, más marcada del oído izquierdo. La otoscopia es normal.
La timpanometría muestra disminución de la compliance y ausencia de reflejo estapedial.
¿Cuál es el diagnóstico más probable?

1) Malformación de la cadena osicular.

2) Hipoacusia neurosensorial hereditaria, de expresión tardía.

3) Otosclerosis.

4) Timpanosclerosis cerrada.

5) Otitis serosa.

Definición

Hipoacusia es la disminución en la capacidad de oír normalmente. Se considera hipoacusia


cuando el umbral de audición en ambos oídos es igual o superior a 25 dB. En el audiograma
existe una clara separación (10 o más dB) de los umbrales entre la vía ósea (normal) y la
aérea (anormal).

Etiología - epidemiología - fisiopatología

Las principales causas de hipoacusia de transmisión son:

 Tapón de cerumen.
 Malformaciones del pabellón auricular.
 Cuerpo extraño en CAE.
 Otoesclerosis
 Otitis Media crónica.
 Otitis Media con efusión.
 Perforación timpánica.
 Fracturas temporales.
 Tumores de oído medio.
Respecto a la Otoesclerosis, esta afecta al hueso de la cápsula ótica provocando hipoacusia
en el 0,4% de la población caucásica.

La hipoacusia de transmisión es aquella en que existe un impedimento de la onda sonora


para alcanzar el oído interno (daño en oído externo o medio). En estos pacientes el estímulo
del vibrador óseo es normal, ya que el oído interno es normal.

Diagnóstico

En general se realiza con anamnesis, el paciente refiere hipoacusia, con pruebas como
Weber y Rinne, y la otoscopia. Con las pruebas de Rinne y Weber se puede distinguir si la
hipoacusia es de transmisión o sensorioneural.

Prueba de Rinne: Consiste en hacer vibrar un diapasón sobre la apófisis mastoides hasta
que la vibración se hace imperceptible, posterior a eso se pone el diapasón frente al oido. Se
considera Rinne positivo si el paciente oye mejor por la vía aérea. Esto puede verse en
personas normales o con hipoacusia de tipo sensorioneural. Se considera Rinne negativo
cuando el paciente oye mejor por vía ósea. Esto es típico de hipoacusia de transmisión.

Prueba de Webber: Se pone el diapasón en vía ósea pero en la línea media (frente, huesos
nasales o incisivos superiores). Dependiendo de dónde escuche mejor el paciente se dará un
diagnóstico. Sonido lateraliza hacia el oído sano: Hipoacusia de percepción. Sonido
lateraliza hacia el lado enfermo: Hipoacusia de transmisión. Se oye igual en ambos lados:
Normal.

Otoscopía: Dependiendo de si es normal o alterada se pueden clasificar los tipos de


hipoacusia de transmisión.

Hipoacusias con Otoscopia Alterada

 Tapón de Cerumen. Se puede presentar como “sensación de oído tapado”,


hipoacusia y tinnitus ipsilaterales. A la otoscopia, se observa una masa café́ o
amarillo oscuro, que ocluye completamente el conducto auditivo externo.
 Otitis Media con efusión: ver capítulo Otitis media con efusión. La
Impedanciometría va a mostrar un timpanograma plano (curva Tipo B).
 Otitis Media Crónica: ver capítulo de otitis media crónica y complicaciones. El
examen con diapasones suele evidenciar una hipoacusia de conducción con un
Weber que lateraliza hacia el lado de la hipoacusia y un Rinne negativo. El
audiograma suele mostrar grados variables de hipoacusia de conducción.

Hipoacusias con Otoscopía Normal

- Otoesclerosis:' Es una enfermedad hereditaria, de origen desconocido, que afecta al tejido


óseo del oído interno (laberinto óseo o cápsula ótica) caracterizada por zonas de rarefacción
y neoformación ósea, con crecimiento de tejido óseo que puede comprometer la ventana
oval y luego la platina del estribo; de este modo puede llegar a fijar la cadena osicular,
produciendo una hipoacusia de conducción. Su síntoma fundamental la hipoacusia, la cual
es lentamente progresiva en años. Se inicia preferentemente al final de la adolescencia, pero
puede aparecer también en la tercera y cuarta década de la vida. La hipoacusia
generalmente se acompaña de tinnitus. La audiometría revela una hipoacusia de
conducción, generalmente bilateral y simétrica, habitualmente con mayor compromiso
auditivo en los tonos graves, y con buena discriminación de la palabra. La
impedanciometría revelará un timpanograma con curva tipo As, complianza estática baja y
ausencia de reflejo acústico. El emabarazo puede acelerar la progresión de la otoesclerosis,
entonces empeora.

Tratamiento

El tratamiento va a depender de la causa de la hipoacusia. Todas requieren tratamiento por


especialista. Para la otoesclerosis el tratamiento de elección quirúrgico con estapedectomía
o estapediotomía.

Seguimiento

El seguimiento es por especialista (ORL).

Diagnóstico diferencial de hipoacusias sensorioneurales


Diagnóstico Sospecha Tratamiento Inicial Seguimiento Derivar

Aspectos esenciales

 La hipoacusia sensorioneural es el déficit sensorial más frecuente de la población


general.
 La presbiacusia es patología GES. Se caracteriza por ser bilateral, simétrica y
progresiva.
 La enfermedad de Meniere presenta la tríada sintomática de hipoacusia, tinnitus y
crisis vertiginosas.

Caso clínico tipo

Paciente masculino 70 año, HTA. Consulta por dificultad al escuchar. Presenta otoscopia
normal. Su conducta sería:

a) Decirle que es normal por su edad y control SOS.


b) Decirle que es normal a su edad y derivar a ORL.

c) Solicitar TAC de cerebro.

d) Descartar cuadro respiratorio alto, dar AINES y controlar en 15 días.

e) Solicitar audiometría e impedanciometría y controlar.

R: b

Definición

La hipoacusia sensorioneural es la incapacidad de oír normalmente a causa de defectos en


el oido interno, la cóclea o el nervio auditivo.

Etiología-epidemiología-fisiopatología

La hipoacusia es el déficit sensorial más frecuente de la población general. La hipoacusia


sensorio neural tiene múltiples causas, entre las que se encuentran:

Presbiacusia Ototóxicos Enf. Meniere Laberintitis Súbita


Neurinoma del acústico Trauma acústico Traumatismo Barotrauma Autoinmune

La pérdida de la audición aumenta su prevalencia con la edad. Desde un 11% entre los 44 y
54 años llegando a la mitad de la población de 75 años. Esto principalmente a causa de
la presbiacusia, una patología multifactorial caracterizada por hipoacusia bilateral simétrica
y progresiva. Dentro de los factores de riesgo se encuentran: Exposición a ruidos,
ototóxicos, bajo NSE, hipertensión, diabetes, tabaquismo, enfermedad vascular, trastornos
inmunológicos y factores hormonales.

Según la Guía GES 2010 de Hipoacusia Neurosensorial en el Prematuro, se estima que


aproximadamente 1-2 de cada 1000 nacidos vivos estaría afectado con hipoacusia
congénita bilateral severa a profunda lo que se extrapolaría a 250 a 500 recién nacidos
afectados con Hipoacusia bilateral congénita.

Diagnóstico

Se realiza con anamnesis, el paciente refiere hipoacusia, más la exploración física que
incluye las pruebas de Weber y Rinne, y la otoscopia. Las pruebas de Rinne y Weber
permiten distinguir si la hipoacusia es de transmisión o sensorioneural. El diagnóstico se
complementa mediante la audiometría. Otros exámenes que pueden realizarse son:
impedanciometría, potenciales auditivos de tronco encefálico y emisiones otoacústicas.

Dependiendo de la causa de la hipoacusia serán los hallazgos en la anamnesis y los


exámenes:
1.- Presbiacusia: Se trata de una hipoacusia producida por la edad; normalmente alrededor
de los 50 años se empieza a producir una pérdida progresiva de la audición. Primero se
manifiesta a través de una pérdida leve para los tonos agudos, para luego ir avanzando en
intensidad y comprometiendo secundariamente a los tonos medios y graves. Se caracteriza
por ser bilateral, simétrica y progresiva y suele acompañarse de tinnitus.

2.- Exposición a Ruido o Trauma Acústico: El oído es incapaz de resistir un trauma


acústico intenso o exposiciones continuadas a ruido de intensidad elevada, sin que ello
implique una lesión orgánica a nivel del órgano de Corti, que se traduce en hipoacusia y
tinnitus. El trauma acústico se puede clasificar en agudo y crónico, según el tiempo de
exposición al ruido.

a) Agudo:exposición a un ruido intenso de corta duración. Puede ser uni o bilateral, y si es


de gran intensidad, puede llevar a la rotura de la membrana timpánica y, en ocasiones a
rotura de la membrana laberíntica, con fístula peri y/o endolinfática.

b) Crónico: La exposición a ruido crónico lleva a un deterioro permanente del umbral


auditivo. La hipoacusia es casi siempre simétrica. Se considera nocivo un ruido continuo de
85 dB en una jornada de 8 horas.

3.- Enfermedad de Meniere: Es una enfermedad del laberinto membranoso, de etiología


desconocida, en la cual histopatológicamente se encuentra un hidrops endolinfático.
Clínicamente se caracteriza por la tríada sintomática de hipoacusia, tinnitus y crisis
vertiginosas recurrenciales. La hipoacusia generalmente es unilateral y habitualmente
es “fluctuante”, se suele acompañar de sensación de “oído tapado”, algiacusia (sensación de
desagrado frente a ruidos intensos) y diplacusia (distorsión auditiva en el oído afectado). El
tinnitus generalmente es unilateral y fluctuante. Las crisis vertiginosas generalmente son
espontáneas de duración variable ( minutos a horas) y generalmente se acompañan de
manifestaciones neurovegetativas.

4.- Laberintitis: El compromiso bacteriano del laberinto ocurre habitualmente por vecindad.
Se clasifican en: laberintitis aguda tóxica (irritación del laberinto por infección meníngea u
ótica sin invasión en oído interno), laberintitis aguda supurada (existe invasión microbiana
del oído interno) y laberintitis crónica (en contexto de OMC).

5.- Hipoacusia Súbita.

6.- Neurinoma del Acústico: Es un tumor benigno, de crecimiento lento, que se forma a
expensas de las células de Schwann de la vaina del nervio vestibular. El primer nervio
comprometido es el coclear, por lo que el tinnitus unilateral es el primer síntoma, con
hipoacusia unilateral leve, hasta déficit neurológico de múltiples nervios craneanos y signos
cerebelosos si hay invasión extensa.

7.- Ototóxicos: Existen varias drogas que pueden producir hipoacusia. Algunos producen
una lesión irreversible, como los aminoglicósidos y los anticancerosos (especialmente los
alquilantes). Otros originan un daño reversible, como la furosemida y el ácido
acetilsalicílico.
8.- Traumatismos: Pueden ir desde la conmoción laberíntica, hasta la fractura de peñasco.

9.- Barotrauma: Esta patología es frecuente en buzos. Los cambios de presión bruscos
pueden llevar a una destrucción de las membranas del oído interno, dando lugar a una
hipoacusia sensorioneural y/o vértigo.

10.- Hipoacusia Autoinmune: Se suele presentar como una hipoacusia sensorioneural


progresiva en semanas o meses, generalmente bilateral y simétrica, que suele llegar a
sorderas profundas.

Tratamiento

El tratamiento va a depender de la causa de la hipoacusia. Todas requieren tratamiento por


especialista.

Seguimiento

El seguimiento es por especialista.

Disfonías agudas

Nivel de manejo del médico general:


Diagnóstico: Específico. Tratamiento: Completo. Seguimiento: Completo.

Aspectos esenciales

 Disfonía de < 15 días de evolución.


 Causa más frecuente es la laringitis aguda.
 Diagnóstico clínico + laringoscopia indirecta.
 Generalmente ceden con reposo vocal, analgésicos y AINES.
 Se debe derivar a especialista si dura más de 2 semanas.

Caso clínico tipo

Paciente consulta por cuadro de 3 días de evolución de disfonía, odinofagia, tos irritativa y carraspera. Al
examen físico faringe congestiva y a la laringoscopia indirecta presenta congestión y edema de las cuerdas
vocales.

Definición

Alteración de una o más de las características acústicas de la voz (timbre, la intensidad y altura tonal) de <
15 días de evolución. La afonía es la alteración máxima, es la pérdida total de la voz.

Etiología-epidemiología-fisiopatología
Las patologías que más habitualmente debutarán con disfonía aguda son, en primer lugar, las inflamaciones
agudas de la laringe. Otros cuadros son las disfonías psicógenas. Además son causa de atención médica de
urgencias los traumatismos laríngeos, que causan, desde una simple disfonía aguda, hasta un compromiso
más o menos grave de la vía aérea que requerirá asistencia urgente.

-Laringitis aguda: Tras un proceso infeccioso respiratorio agudo, se presenta la laringitis aguda más
frecuentemente de etiología viral que suele provocar disfonía, sensación de prurito, tos seca y carraspera. Es
autolimitado aproximadamente por siete días, máximo dos semanas. Sin embargo, en pacientes con tos
persistente o abuso (o mal uso) de la voz, puede desencadenar una disfonía disfuncional. También puede
preceder a una parálisis laringea. Cuando es bacteriana corresponde a: Hemophilus influenzae, Estreptococo
Beta hemolítico o Estafilococo.

-Traumatismo laríngeo: Los traumatismos cervicales, pueden afectar la voz y la función respiratoria. Pueden
ocasionar fracturas laríngeas, dislocación del aritenoides, parálisis nervio laringeo recurrente y estenosis
laringea. La intubación endotraqueal, aunque sea breve, es una posible causa de traumatismos laríngeos.

-Disfonías funcionales: Son condicionadas por abuso o mal uso de la voz. Pueden presentarse inicialmente
de forma intermitente y hacerse cada vez más frecuentes, hasta ser continuas. También pueden presentarse
de forma aguda o siendo precedida por un episodio de laringitis viral aguda.

-Parálisis laríngeas: La caracteriza la voz bitonal. Puede ser de causa traumática, quirúrgica (especialmente
cirugía tiroidea y mediastínica), vascular, tumoral o idiopática.

-Disfonías psicógenas: Es la alteración de la voz producida por un trastorno psicológico o psiquiátrico. Son
principalmente neurosis de conversión, que corresponde a una reacción de defensa ante el mundo exterior.
La forma más frecuente es la afonía completa.

-Nódulos de cuerda vocal: Pequeña formación redondeada que afecta a una o ambas cuerdas vocales e
impide que su cierre sea completo, generando hiatus, pérdida de aire y disfonía concomitante. Típico de las
mujeres que trabajan con la voz, en edad joven y media. Se manifiesta por disfonía que no dura más de 15
días.

Diagnóstico

Clínica: En la anamnesis indagar temporalidad, forma de aparición, duración y factores agravantes y/o
atenuantes, síntomas acompañantes. Antecedentes médicos (pulmonares, RGE, neurológicos, rinitis,
radioterapia, etc). Antecedentes quirúrgicos y/o traumáticos (intubación, cirugía de cuello, trauma laringeo,
etc).

Examen físico: Debe iniciarse describiendo las características de la voz: una voz grave, rasposa puede
sugerir laringitis o una masa en la laringe, si se percibe una voz débil, con excesivo escape de aire durante la
fonación puede indicar paralisis cordal o atrofia. Examinar faringe y cuello, en busca de adenopatías,
tumores cervicales y patología tiroidea. No olvidar nunca palpar el cuello. La laringoscopia indirecta suele ser
realizada por el ORL. Nasofibroscopia y/o Laringoscopia directa.

Laringitis aguda: Síntomas: Disfonía, odinofonía (dolor al hablar), disfagia, odinofagia, tos irritativa y
síntomas de obstrucción respiratoria alta (niño). En Laringoscopía indirecta, directa o fibroscopía, podría
observarse ocasionalmente, congestión y edema de las cuerdas vocales, con o sin inflamación del resto de la
mucosa endolaríngea, generalmente, asociada a epiglotitis, edema faríngeo y/o traqueal vecino.

Nódulos Laríngeos: En la nasofibroscopía se visualiza una lesión de tipo nodular en el 1/3 anterior de las
cuerdas vocales. Habitualmente bilateral

Tratamiento

Laringitis aguda: Un gran porcentaje suele ser de regresión espontánea, fundamentalmente en las de
etiología viral. En estos casos es útil un tratamiento sintomático (analgésicos, antiinflamatorios no
esteroidales y descongestionantes), además de reposo vocal, supresión de noxas (tabaco, inhalantes
químicos, etc.) y humidificación del aire inspirado. Uso de antibióticos solo en caso de sobreinfección
bacteriana. En caso de pacientes con síndrome obstructivo alto, fundamentalmente en el niño, puede
indicarse epinefrina racémica (paciente hospitalizado), corticoides (dexametasona) y/o intubación (o
traqueostomía), según se requiera.

Nódulos Laríngeos: Tratamiento Foniátrico (DERIVAR). Si persiste: Microcirugía.

Seguimiento

En caso de un laringitis aguda, si los síntomas desaparecen, no se requiere seguimiento. No obtante, si no


ceden y se prolongan por más de 2-3 semanas, se deriva a especialista ORL. Además, en un paciente
fumador, toda disfonía de > de 15 días de evolución debe considerarse cáncer hasta demostrar lo
contrario.

Disfonías crónicas
Nivel de manejo del médico general: Diagnóstico: Sospecha. Tratamiento: Inicial. Seguimiento: Derivar.

Aspectos esenciales

 Constituye un signo y no una patología en sí misma.


 El sexo femenino y el hábito tabáquico son factores de riesgo.
 La laringoscopia es un procedimiento diagnostico fundamental.

Caso clínico tipo

Una mujer casada de 35 años y madre de cuatro hijos pequeños, consulta por disfonía de tres meses de
evolución que no mejora con mucolíticos. La palpación cervical es negativa. Es probable que se trate de
nódulos vocales

Definición

Alteración de una o más de las características acústicas de la voz (timbre, la intensidad y altura tonal) de >
2 semanas de evolución.

Etiología-epidemiología-fisiopatología

Las disfonías crónicas pueden ser clasificadas en: 1) orgánicas; 2) funcionales; y 3) orgánicas de base
funcional. En las primeras, se afecta estructuralmente el órgano laríngeo por patologías propias de él o por
repercusión de la afectación de estructuras y órganos vecinos. En las segundas, no hay afectación anatómica
laríngea, pero sí un funcionamiento inadecuado del mecanismo fonatorio, respiratorio o resonancial. Las
últimas corresponden a afectación de estructuras laríngeas por mal uso vocal.

Causas:

Disfonías funcionales: no hay alteración orgánica demostrable.

 Técnica vocal inapropiada: mal uso y abuso vocal.


 Factores psicológicos involucrados (personalidad predispuesta)

Disfonía orgánica de base funcional: debidas a una mala técnica vocal. Son lesiones benignas.

 Pólipos: debido a trauma vocal. Disfonía en grado variable dependiendo de tamaño y ubicación.
 Nódulos: directamente relacionado con mala técnica vocal. Disfonía en tonos graves. Causa más
frecuente de disfonía en mujeres y niños.
 Edema de Reinke: disfonía en tonos graves. Más frecuente en mujeres, fumadoras. Factores
etiopatogénicos: tabaco (+ importante), RGE, hipotiroidismo.
 Úlceras de contacto, granulomas: sensación de cuerpo extaño, carraspera, tos, dolor en borde
superior del tiroides. Etiología: RGE (+ importante), abuso vocal, intubación.
 Laringitis crónica: inflamación persistente mucosa laríngea. Síntomas: ronquera, tono vocal grave,
fatiga vocal.

Disfonía orgánica:

 Papilomatosis laríngea. Etiología VPH cepas 6, 11, 16 y 18.


 Congénitas: disgenesias congénitas de la laringe (hipoplasias, sulcus, membranas intercordales,
etc.)
 Inflamatorias: laringitis aguda (infecciosa, irritativa [RGE, polución, tabaquismo], traumática),
laringitis crónica (excluyendo las de base funcional), post-irradiación cervical.
 Postquirúrgicas y traumáticas. Especialmente cirugía de tiroides.
 Tumores benignos (de causa no funcional): papilomas, laringoceles, etc.
 Neurológicas: parálisis cordal (centrales y periféricas).
 Endocrinológicas
 Involutivas (secundarias a la edad)
 Farmacológicas

Diagnóstico

De estudio por el especialista, la labor del médico general será sospechar causas malignas de disfonía para
su derivación de urgencia. Los síntomas que sugieren malignidad son: dificultad para respirar, tos, estridor,
hemoptisis, disfagia, odinofagia y pérdida de peso, que no estén asociados a una infección respiratoria
aguda. Puesto que la disfonía constituye un signo y no una patología en sí misma, el enfoque diagnóstico
debe centrarse en el estudio etiológico, para lo cual son fundamentales la historia clínica y los métodos
exploratorios del aparato vocal (por especialista) tales como: laringoscopía indirecta, fibras ópticas: rígidas y
flexibles (nasofibroscopía), estroboscopia, laboratorio de voz, electromiografía laríngea, evaluación
perceptivo acústica de la voz. Imagen: TAC.

Tratamiento

Por especialista. Dependerá de la etiología. Es multidisciplinario. Se puede generalizar en terapia


fonoaudiológica para las patologías funcionales y terapia medico-quirurgica para patologías orgánicas. Es
fundamental eliminar los agentes causales modificables (tabaco, RGE, infecciones respiratorias) y enseñar
hábitos de buena higiene vocal.

Seguimiento

Por parte del especialista.

Drenaje de hematoma nasoseptal


Nivel de manejo del médico general: Realizar

Introducción

Es un procedimiento de urgencia ante la presencia de hematoma nasoseptal, que debe ser


sospechado en todo trauma nasal, cuyo diagnóstico se realiza mediante inspección de las
narinas, observándose abombamiento de la línea media bilateral. La intervención rápida de
esta condición permite preservar la irrigación del cartílago septal, previniendo sus
complicaciones como son la perforación por necrosis, sobreinfección y formación de
abscesos.

Indicaciones

 Diagnóstico de hematoma nasoseptal

Contraindicaciones

 No existen contraindicaciones absolutas

Complicaciones

 Abscesos septales
 Infecciones contiguas; celulitis orbitaria, meningitis
 Trombosis de seno cavernoso

Materiales e insumos

 Espéculo nasal con fuente de luz adecuada


 Pinza bayoneta
 Anestesia local
 Gazas, guantes estériles, apósitos, tiras de algodón estériles, cinta adhesiva, drenaje
penrose
 profilaxis antibiótica ej cloranfenicol ungüento
 Solución fisiológica
 Aguja 18-20 gauge
 Bisturí n° 11
 Aspiración
 Profilaxis antibiótica (amoxicilina + ácido clavulánico)

Preparación y procedimiento

1. Sentar al paciente en sillón con respaldo alto


2. Colocar un lienzo o bata que cubra cuello y región anterior de tórax
3. Se aplica anestesia local
4. Localizado el hematoma se procede a realizar una incisión a lo largo de la base del
hematoma, permitiendo la salida del contenido apoyando con aspiración e irrigando
con solución salina
5. Una vez drenado se procede a introducir las tiras de algodón embebidas en
ungüento antibiótico de manera escalonada para tamponar por anterior, procurando
no generar mucha presión dentro de la cavidad nasal
6. Finalmente se coloca un apósito en la base de la nariz con cinta adhesiva
7. Reexplorar 2-3 días después.
Epistaxis a repetición
Nivel de manejo del Medico General: Diagnostico: Especifico Tratamiento: Completo
Seguimiento: Derivar

Aspectos esenciales

 Frecuente en niños.
 La mayoría de los casos son leves y autolimitados, pero hay un porcentaje de casos
graves con anemia secundaria.
 Siempre estudiar la causa.
 Descartar alteración de la hemostasis.
 Tratamiento según etiología.

Caso clínico tipo

Paciente de 5 años, consulta por epistaxis en múltiples oportunidades, de baja cuantía y


autolimitadas. Desde los 3 años de edad. Al examen físico no se observa sitio de sangrado
activo, con signos vitales estables.

Definición

Pérdida de sangre proveniente de los vasos de las fosas nasales. En episodios repetidos.

Etiología-epidemiología-fisiopatología

Motivo de consulta frecuente en la población general. Con una prevalencia de 10 a 15%.


60% de la población ha presentado epistaxis y un 6 % ha requerido tratamiento médico.

Lo más frecuente es que el origen de la epistaxis sea anterior proveniente del plexo de
Kiesselbach (90%). Con menor frecuencia son posteriores provenientes principalmente de
la ruptura de la arteria esfenopalatina.

Causas:

Locales Sistémicas
Traumáticas.
Arterioesclerosis - HTA
90% digitales, por grataje. También trauma
directo y barotrauma.
Inflamatorias.
Enf. Infecciosas: (Ej. Hepatitis, influenza,
Rinitis aguda (Viral o bacteriana) y Crónica MNI).
(alérgica).
Defectos anatómicos.
Vasculares - Traumatismos vasculares.
(Púrpuras, hepatopatías, Rendu Weber-Osler)
Ej. Desviación septal
Quirúrgicas Hematológicas o intravasculares.
Tumorales Endocrinológicas

Diagnóstico

Se debe tratar como un signo en el manejo clínico inmediato, y como un diagnóstico en el


mediato. Se debe tratar adecuadamente el sangrado local, así como tratar la etiología y dar
un soporte hemodinámico adecuado.

Anamnesis: Es de vital importancia y no es reemplazable por exámenes de laboratorio.


Determinar las características de la epistaxis a repetición, frecuencia, duración, intensidad
(cantidad de sangre perdida), edad de inicio, y si se trata de sangramiento uni o bilateral; ya
que en epistaxis leves sólo existe 10% de anormalidades de laboratorio, que aumentan a
58% en severas a repetición según algunos estudios. Investigar en forma dirigida historia de
petequias, púrpura, equimosis de fácil aparición, hemorragias quirúrgicas o alveolorragias
posteriores a extracciones dentarias, así como antecedentes de síndrome de mala absorción
y de ingesta de medicamentos antiplaquetarios. Las hemorragias recurrentes en cuya
investigación no se ha encontrado una causa local que las explique, hace también sospechar
una alteración de hemostasis.

Examen físico completo: Presión arterial - pulso. Piel: Inspeccionar si presenta equímosis,
petequias, hematomas, etc. Adenopatías.

Examen oral y faríngeo:Si hay sangramiento posterior (observación de descenso de sangre


fresca por la orofaringe, en posición sentado) telangiectasias, petequias, etc.

Examen nasal: Buscar dirigidamente un cuerpo extraño o una masa nasal que explique la
epistaxis a repetición.

Examenes de laboratorio: Hemograma, VHS, Pruebas de coagulación.

Imágenes:si se sospecha tumor. TAC, RNM

Tratamiento

En el manejo agudo de la epistaxis: (ver con más detalle en tema epistaxis leve).

1. Determinar la cuantía del sangrado y reanimación hemodinámica inmediata si es


necesario.
2. Identificación del punto sangrante (idealmente).

3. Control terapéutico mediante medidas locales, invasivas u otras.

La recurrencia de la epistaxis se puede prevenir solamente considerando las múltiples


causas con tal de aplicar medidas preventivas apropiadas. Además, se recomienda la
humectación de la mucosa nasal y prevención del trauma local.

Otras alternativas terapéuticas son la cauterización, corticoides nasales en pacientes con


rinitis o uso de descongestionanes nasales con anti histamínicos ( en mayores de 2 años).

Seguimiento

Por especialista.

Epistaxis Leve
Nivel de Manejo de Medico General:
Diagnóstico: Específico Tratamiento: Completo Seguimiento: Completo

Aspectos esenciales

 Es un signo, por lo tanto se debe buscar la causa.


 Sin compromiso hemodinámico.
 Mayor frecuencia durante la infancia.
 El 90% provienen de la zona de Kiesselbach.
 La causa más frecuente es el grataje.
 La mayoría autolimitadas.

Caso clínico tipo

Paciente de 5 años con hemorragia nasal escasa, que acude a control, con signos vitales
estables, sin obstrucción de vía aérea.

Definición

Hemorragia proveniente de la fosa nasal, con vía aérea permeable y signos vitales estables.

Etiología-epidemiología-fisiopatología
Afección muy frecuente. Sólo el 6-10% tiene compromiso hemodinámico o no son
autolimitados por lo que motivan consulta. Su mayor frecuencia se presenta durante
lainfancia, es rara en recién nacidos y lactantes. Se clasifican en:

- Anteriores: 90%, sitio identificable, menos profusa. Provienen del Plexo


deKiesselbach(mucosa delgada, expuesta a estímulos mecánicos como grataje y aire).

- Posteriores:10%, sitio no identificable, más profusa. Provienen de la


Arteria Esfenopalatina.

Causas:

Locales Sistémicas
Traumáticas.
Arterioesclerosis - HTA
90% digitales, por grataje. También
trauma directo y barotrauma.
Inflamatorias.
Enf. Infecciosas: (Ej. Hepatitis, influenza, MNI).
Rinitis aguda (Viral o bacteriana) y
Crónica (alérgica).
Defectos anatómicos.
Vasculares - Traumatismos vasculares. (Púrpuras,
hepatopatías, Rendu Weber-Osler)
Ej. Desviación septal
Quirúrgicas Hematológicas o intravasculares.
Tumorales Endocrinológicas

Diagnóstico

1. Se debe tratar como signo en el tratamiento agudo inmediato, y como un diagnóstico en


el enfoque mediato.

2. Anamnesis debe ser exhaustiva, completa:

 Lado que comenzó la hemorragia.


 Hemorrragia aislada o escupe mucha sangre.
 Duración del sangrado. Estimación de la cantidad de sangre perdida.
 Si tiene resfrío actual o rinitis
 HTA
 Fármacos: Aspirina, TAC, AINE, Isotretionina.
 Historia previa de epistaxis.
 Enfermedades predisponentes.
 Hábitos (en niños).
3. Examen físico completo: Presión arterial - pulso. Piel: Inspeccionar si presenta
equímosis, petequias, hematomas, etc. Adenopatías.

Examen oral y faríngeo: Si hay sangramiento posterior (observación de descenso de sangre


fresca por la orofaringe, en posición sentado) telangiectasias, petequias, etc.

Tratamiento

Generalmente cede concompresión mantenidasobre la región septal anterior. Si esto no


ocurre, y si se identifica punto sangrante, previaanestesia con algodón embebido en
Lidocaína 2 % o 4%, se procede a cauterizar con perlas de nitrato de plata. Si el
compromiso es extenso, no se identifica el punto sangrante o no cede con la perla, se
procede ataponamiento anterior: Lo común es realizarlo con gasas o algodón, este material
debe estar lubricado (vaselina o ungüento antibiótico si está disponible), también se pueden
usar esponjas expansibles. Insistir en la buena preparación con anestésico tópico,
vasoconstrictores, tener una buena luz, en lo posible un equipo de aspiración. Se debe
cubrir gran parte de la fosa nasal (10 cms en adulto), colocar de tal forma que produzca
presión. Se mantiene po r3-5 días, generalmente con uso concomitante deATB
orales(amoxicilina). En caso de epistaxis posterior: taponamiento posterior,
HOSPITALIZAR.

Seguimiento

Generalmente no necesitan mayor estudio. Control en 3-5 días para retirar taponamiento y
observar evolución. Si luego de este control el sangrado ha cedido, control SOS.

Epistaxis masiva
Nivel de manejo del médico general: Diagnóstico Específico Tratamiento
Completo Seguimiento Derivar

Aspectos esenciales

Cursa con compromiso hemodinámico.

Mayor frecuencia de epistaxis posterior en adultos mayores.

Manejo orientado a sostén hemodinámico, siempre clasificar grupo sanguíneo y RH.

Caso clínico tipo

Paciente de sexo masculino, 62 años ingresa al SU debido a epistaxis de gran cuantía,


bilateral. Al examen físico destaca facie pálida, PA 98/56, FC 150 lpm.
Definición

Hemorragia proveniente de los vasos de las fosas nasales, que cursa con compromiso
hemodinámico.

Etiología-epidemiología-fisiopatología

La epistaxis se produce principalmente en la región septal anterior, aunque en adultos


mayores aumenta la frecuencia de sangrados posteriores. En general la nariz recibe
irrigación tanto del sistema carotídeo interno como externo. La zona anterior del tabique
ricamente irrigada se conoce como plexo de Kiesselbach, zona donde se producen la
mayoría de la epistaxis.

Etiología: Se sistematiza clásicamente en causas locales y sistémicas.

Locales: traumatismos, grataje, inflamaciones (ej: rinitis alérgica), deformaciones del


tabique (se pierde el flujo laminar y aumenta el roce), cuerpos extraños, enfermedades
granulomatosas, irritación químicos y tumores.

Sistémicas: Hipertensión arterial, discrasias sanguíneas (son de mayor cuantía y


prolongadas, ej: enf. Von Willebrand, hemofilias y tratamiento anticoagulante),
alteraciones vasculares, medicamentos, infecciones, alteraciones cardiovasculares, IRC e
Insuficiencia hepática. Dentro del tratamiento de la epistaxis se debe considerar el
tratamiento de la enfermedad de base.

Dentro de las complicaciones de la epistaxis masiva encontramos: Shock hipovolémico,


aspiración de sangre en pacientes con reflejos disminuidos, alterados o demasiado sedados,
elevación de la uremia por ingestión masiva de sangre, infarto cardíaco, insuficiencia renal,
accidente vascular encefálico.

Diagnóstico

En el análisis clínico se debe buscar su causa o factor desencadenante, se debe tratar


adecuadamente el sangrado local, así como tratar la etiología y dar un soporte
hemodinámico adecuado.

Historia clínica: una historia clara y breve, al mismo tiempo que se examina y se define
conducta; dirigir la anamnesis hacia reconocer si es una epistaxis anterior o posterior,
aislada en el tiempo o recurrente, si hay algún factor desencadenante local o sistémico.

Examen físico: Siempre vigilar de forma seriada los signos vitales. Localizar el o los puntos
sangrantes. Los cuadros hemorrágicos importantes, generalmente se presentan en forma
bilateral, con sangrado posterior, el estado general del paciente esta alterado, fascie pálida,
con compromiso hemodinámico, el examen otorrinolaringológico es difícil, muchas veces
estos pacientes son los que deben hospitalizarse.
Tratamiento

Un enfermo con epistaxis severa es un enfermo grave, sobretodo si es un adulto mayor por
lo que debe ser:

1. Hospitalizado.

2. Realizar un estricto control de signos vitales.

3. Efectuarle hematocrito diariamente, incluso más de una vez de ser necesario


dependiendo de la cuantía del sangrado y del descenso de éste.

4. Monitorizarlo.

5. Poseer siempre una vía sanguínea permeable.

6. Transfundirlo en caso de Shock, o hematocrito bajo 20.

7. Determinar la uremia diariamente, por la ingestión masiva de sangre por vía digestiva.

Como manejo inicial por médico general realizar taponamiento anterior y posterior. Si el
sangrado no cede efectuar manejo por especialista.

Tratamientos quirúrgicos: Ligaduras arteriales, septoplastía, embolización.

Seguimiento

Por especialista.

Estridor Laringeo
Nivel de manejo del médico general: Diagnóstico: Especifico Tratamiento: Inicial
Seguimiento: Derivar

Aspectos esenciales
Constituye un signo y no una patología en sí.
La población pediátrica es la más frecuentemente presenta estridor laríngeo.
Laringomalacia es la causa de estridor crónico más frecuente en el lactante y RN.

Caso clínico tipo


Consulta a Ud. una madre con su hijo de 12 meses porque siente que tiene un “ruido al
respirar”, que lo siente desde que nació, a la anamnesis dirigida el llanto es sin disfonía, el
estridor aumenta en decúbito dorsal o cuando el niño se alimenta. Al examen físico se
constata estridor inspiratorio y no se encuentran signos de dificultad respiratoria.
Definición
Sonido agudo y monofónico al respirar originado por obstrucción de la corriente de aire en
la vía aérea superior.

Etiología-epidemiología-fisiopatología
Traduce un fenómeno fisiopatológico obstructivo, pero no representa un diagnóstico por sí
mismo, lo que obliga siempre a realizar un estudio etiológico. Orientación anatómica
dependiendo de la fase del ciclo respiratorio que comprometa:

Estridor inspiratorio: alteración supraglótica o estructuras más altas como naso u


orofaringe.
Estridor espiratorio: lesiones traqueales bajas o traqueobronquiales (intratorácicas).
Estridor bifásico: lesiones glóticas, subglóticas o traqueales altas.
Más frecuente en niños, debido a las características anatómicas de la vía aérea en este
período.

Causas:

Congénitas: laringomalacia, parálisis de cuerdas vocales, hemangioma subglótico, estenosis


subglótica congénita, membranas laríngeas, anillos vasculares.
Infecciosas: crup laríngeo, epiglotitis, bronquitis, amigdalitis severa, absceso retrofaríngeo.
Traumáticas: aspiración de cuerpo extraño, intubación prolongada.
Extralaríngeas: malformaciones cráneo faciales, atresia parcial de coanas, macroglosia,
quiste del conducto nasolagrimal, hipertrofia del anillo de Waldayer, etc.
Dentro de las causas congénitas, la más frecuente es la laringomalacia (estridor laríngeo
congénito, 60-70%). Provoca estridor en el recién nacido y lactante menor. Existe una
inmadurez del esqueleto cartilaginoso de la laringe, que se hace más evidente en la
supraglotis, por lo que el esfuerzo inspiratorio produce que la epiglotis, los aritenoides, y
los repliegues aritenoepiglóticos, se movilicen hacia el lumen de la laringe (colapso),
produciendo un estridor de tipo inspiratorio. Ocasionalmente estas estructuras, son
inspiradas hacia la glotis, colapsando la vía aérea. En general, el llanto es sin disfonía, el
estridor aumenta en decúbito dorsal o cuando el niño se alimenta. El reflujo gastroesofágico
puede exacerbar el síntoma en esta patología así como en otras causas de estridor. Más del
90% de los casos no requiere de ningún tratamiento. En un pequeño número se debe
efectuar resección parcial de la epiglotis y de los repliegues aritenoepiglóticos con cirugía
convencional o con láser. En situaciones extremas se debe realizar traqueostomía.

Diagnóstico
Anamnesis detallada. Considerar antecedentes perinatales y edad de inicio. Por lo general
los trastornos congénitos (Ej. Laringomalacia) se presentan en las primeras semanas de
vida. La aspiración de cuerpo extraño se empieza a ver desde los 6 meses de edad y es más
frecuente en torno a los 2 - 3 años. Crup es más frecuente en niños entre 6 y 36 meses de
edad y es muy raro en mayores de 6 años. Tener en cuenta además la velocidad de
presentación. Un inicio abrupto y sin otra sintomatología hace sospechar aspiración de
cuerpo extraño. Las reacciones alérgicas severas también tienen un inicio abrupto. Por otro
lado estridor acompañado de fiebre, ronquera y otros síntomas respiratorios hace sospechar
una causa infecciosa.
Examen físico. Lo primero es descartar si el paciente presenta obstrucción de la vía aérea.
Si el paciente está estable y no requiere una intervención inmediata, entonces proceder a
una exploración física completa.

Exámenes: Tomar exámenes de laboratorio si se sospecha causa infecciosa, radiografía de


cuello si se sospecha absceso retrofaríngeo, crup o cuerpo extraño. La visualización de la
vía aérea se realiza en estridor crónico o cuando se sospechen causas congénitas (por
especialista).

Tratamiento
Manejo de la vía aérea en casos graves. Manejo de causas infecciosas (ver capítulo de
laringitis). Las causas congénitas son de resorte del especialista, quien decidirá el
tratamiento según la etiología. Para la laringomalacia, en un 90% no requerirá tratamiento
(resolución espontánea ocurre antes de los 18 meses); el porcentaje restante, así como la
estenosis subglótica congénita y las membranas laríngeas son de resolución quirúrgica.

Seguimiento

De responsabilidad del especialista, es importante en patologías como la laringomalacia y


el hemangioma subglotico en los cuales se espera resolución espontánea.

Fractura nasal reciente


Nivel de manejo del médico general: Diagnóstico Especifico Tratamiento Inicial
Seguimiento Derivar

Aspectos esenciales

Son las fracturas faciales más frecuentes. Corresponde al 50% de las fracturas
maxilofaciales.

En un primer momento se debe estar atento a complicaciones como el hematoma septal,


fístula de LCF.

El tratamiento definitivo es la reducción cerrada, hasta 2 semanas después.

Mal manejo lleva a consecuencias estéticas y funcionales.

Caso clínico tipo

Paciente consulta de madruga al servicio de urgencia luego de ser asaltado, relata haber
recibido varios golpes de pies y puños. A la inspección se aprecia el tercio medio facial
edematizado, leve desviación de la nariz y epistaxis. A la palpación esta resulta dolorosa,
con presencia de un resalte óseo y movilidad anormal.
Definición

Es la “solución de continuidad” de las estructuras que conforman la pirámide nasal, la cual


ocasiona daño estructural con consecuencias funcionales y/o estéticas. Puede afectar al
puente nasal, pared lateral, y/o el tabique nasal.

Etiología - epidemiología - fisiopatología

Son las fracturas maxilofaciales más frecuentes, las causas más comunes son agresiones
físicas, accidentes de tránsito, lesiones deportivas y caídas casuales. Es más frecuente en
hombres que en mujeres. Los tipos de traumatismo en orden de frecuencia son: lateral,
frontal e inferior.

Lateral: Hundimiento de uno o ambos huesos. Desplazamiento de la pirámide.


Desplazamiento y/o desviación septal. Laterorrinia.

Frontal: Fractura simple huesos propios. Aplanamiento nasal y ensanchamiento. Separación


de huesos propios en línea media. Separación de cartílagos laterales de huesos propios.
Fracturas asociadas.

Inferior: Fracturas septales y luxación cartílago cuadrangular.

El septum nasal es una zona con excelente irrigación y cicatrización. 1/3 superior rígido.
2/3 superiores flexibles.

En niños siempre sospechar maltrato infantil

Diagnóstico

El diagnóstico es clínico (y muchas veces evidente) frente al antecedente del trauma y la


presencia de dolor, desviación, obstrucción respiratoria, epistaxis, resaltes o espículas
óseas, laterorrinia, crepitación, fragmentos angulados, enfisema subcutáneo y el
hundimiento de la pirámide ósea al realizar la palpación. Es importante consignar la forma
nasal previa (pedir foto o carnet), mecanismo, alcohol, drogas y compromiso de conciencia.

Examen físico: Siempre realizar una especuloscopía nasal anterior para descartar un
hematoma septal que debe ser drenado inmediatamente (masa dolorosa en la porción
anteroinferior del septum) y para evaluar desviaciones antiguas o postraumáticas. En la
inspección externa evaluar deformidad, hematomas, edema palpebral, hemorragia
subconjuntival, asimetría facial y estrabismo.

Palpación: Movilidad nasal, crepitación, depresión, desplazamiento, dolor, enfisema


subcutáneo, reborde orbitario y arcos zigomáticos. Son signos importantes de buscar la
rinorraquia (indicadora de fistula de LCR), epistaxis persistente pese al taponamiento
anterior (sugiere lesión de la arteria etmoidal anterior).
Radiología: Radiografías son de escasa utilidad ya que tiene muchos falsos positivos y
negativos. Realizar Rx en proyecciones de Waters y Huesos propios bilateral (aspecto
medico legal). Si se quiere descartar otras lesiones. Realizar TAC.

Tratamiento

Las intervenciones iniciales (luego del diagnóstico) van dirigidas a la búsqueda de


complicaciones como el hematoma septal y fístula de LCR, control de la epistaxis, manejo
del dolor y del proceso inflamatorio asociado. Si se presenta un hematoma septal se tiene 3
días como plazo máximo para manejo. Efectuar drenaje y administrar antibióticos.

Derivar a ORL para reducción de la fractura. La reducción debe realizarse posterior a la


aparición del edema y puede diferirse dentro de los primeros 12 días en el adulto y dentro
de la primera semana para niños.

Seguimiento

Por especialista. Es importante controlar la evolución de la fractura y de las posibles


complicaciones. En general, el porcentaje de deformidad posterior al trauma es del 15 al
45%, y aumenta cuando se asocia a lesión de tabique nasal.

Hematoma nasoseptal
Nivel de manejo del médico general: Diagnóstico Específico Tratamiento
Completo Seguimiento Derivar

Aspectos esenciales

Ocurre complicación del trauma maxilofacial o nasal.

El diagnóstico es clínico.

El tratamiento debe ser precoz con drenaje y antibióticos para evitar complicaciones (3 días
como plazo máximo).

Las principales complicaciones son la infección y la tromboflebitis.

Caso clínico tipo

Paciente de 35 años recibe un golpe nasal contuso, evoluciona con dificultad respiratoria
nasal, abombamiento bilateral del tabique, asociado a dolor de la zona.
Definición

Hematoma que se produce entre el cartílago septal y el mucopericondrio que está sobre él,
tras un traumatismo nasal.

Etiología-epidemiología-fisiopatología

Un traumatismo nasal contuso y de impacto tangencial, puede producir un desprendimiento


mucopericóndrico de la porción cartilaginosa del tabique, desde detrás de la columela hasta
las porciones óseas. De esta manera aparece un hematoma ocupando ese espacio, a menudo
de manera bilateral. Si no se actúa con rapidez, puede infectarse (habitualmente por
estafilococo) y abscedarse.

Se ha reportado una incidencia de 0,8 a 1,6% en pacientes examinados por un especialista.


Pero es una entidad que se subdiagnostica, hasta que ocurren las complicaciones.

Es más frecuente en niños que en adultos.

Si no se trata puede conducir a necrosis septal dentro de 72 a 96 horas, lo que es


irreversible. Esto puede causar el hundimiento del dorso de la nariz (nariz en silla de
montar).

Diagnóstico

Historia clínica: Los síntomas son inespecíficos, pero lo más común es que se presente con
obstrucción, dolor, rinorrea y fiebre. El hematoma septal se puede desarrollar incluso con
un trauma pequeño. Por lo que se debe tener un alto índice de sospecha. Se debe descartar
hematoma en todo paciente con antecedente de trauma nasal, aunque sea un trauma de baja
intensidad.

Examen físico: Realizar especuloscopía nasal. En caso de infección, aparecen intensos


dolores, cefalea, fiebre y enrojecimiento del dorso nasal.

Tratamiento

Requiere evacuación del hematoma mediante incisiones verticales del mucopericondrio,


con bisturí frío, a 1 cm del borde caudal del tabique. Si es bilateral tener cuidado con que
no coincidan las incisiones por riesgo de perforación septal. Posteriormente se realiza un
taponamiento con el objetivo de aproximar el pericondrio al cartílago, y de esta forma
garantizar su nutrición. Dar antibióticos de amplio espectro profilácticos. Los antibióticos
deben curbir: Staphylococcus aureus, Streptococcus pneumoniae, SBHGA y Haemophilus
influenzae.

Complicaciones: Si se compromete la nutrición del septum, en un plazo de 24 a 48 horas,


pueden producirse necrosis e infección que conllevarían a la pérdida del soporte nasal
dorsal, con la aparición de una nariz en silla de montar. Las venas del tabique nasal drenan
directamente al seno cavernoso, por lo que un absceso dejado a su evolución puede originar
émbolos sépticos, con la aparición de infecciones intracraneales y trombosis de los senos.

Seguimiento

Por especialista

Hipoacusia neonatal
Nivel de manejo del médico general: Diagnóstico Sospecha Tratamiento Inicial
Seguimiento Derivar

Aspectos esenciales

 El diagnóstico temprano es de vital importancia para el pronóstico.


 El 50% de los RN sordos no tiene factores de riesgo.
 El tamizaje se realiza con los potenciales auditivos de tronco encefálico.

Caso clínico tipo

Se realizará tamizaje, según GES a:

 Todo RN
 Todo RN con factores de riesgo
 RN prematuros < 32 semanas y/o 1500grs.
 A quienes opten a él.

Definición

Disminución congénita de la agudeza auditiva, reflejada en un aumento del umbral


acústico.

Etiología-epidemiología-fisiopatología

La incidencia es de 1-2 x 1000 nacidos vivos.

Enfermedad que se debe diagnosticar en los primeros meses de vida. Tratamiento con
audífonos debería ser implementado a los 6 meses de vida.

Factores de riesgo para sordera congénita:

 Más de 48 horas en UCI neonatal.


 Infecciones neonatales (CMV, Toxoplasmosis, sífilis, rubeola, virus herpes).
 Alteraciones craneofaciales.
 Peso menor a 2 kg.
 Prematurez.
 Hiperbilirrubinemia.
 Historia familiar de hipoacusia sensorioneural.
 Síndromes asociados a hipoacusia.
 Uso de ototóxicos.

Diagnóstico

En Chile, según guía GES se pesquisa hipoacusia en:

 RN menor de 1500 gramos.


 Menor de 32 semanas.

Para el tamizado se utiliza un protocolo de potenciales auditivos de tronco encefálico


abreviado.

Existen 3 formas para detectar la hipoacusia congénita en el RN:

1. Derivación espontánea por el pediatra.


2. Tamizaje en RN con FR para hipoacusia.
3. Tamizaje universal en RN.

El tamizaje de RN con FR es el actualmente utilizado por el plan AUGE en Chile. Tienen


una incidencia 10 a 20 veces mayor que niños sin FR, pero sólo un 50% de los niños con
sordera congénita presenta algún factor de riesgo.

Los métodos que se utilizan para detectar sordera en RN son:

 Emisiones otoacústicas.
 Potenciales auditivos de tronco encefálico.
 Audiometría condicionada (en mayor de 6 meses).

Tratamiento

De responsabilidad del especialista, está basado en:

 Implementación precoz de audífonos bilaterales: la sordera no es absoluta, quedan


restos auditivos que hay que potenciar antes de los 5 años para el desarrollo del
lenguaje. Meta de los audífonos: llegar a la laguna gris en la audiometría para poder
desarrollar el lenguaje (entre 10 y 50 db)
 Rehabilitacion fonoaudiologica
 Re-evaluación de la audición.
 Implante coclear: la FDA lo aprueba después de un año de vida. Reemplaza el OI.

Seguimiento
De responsabilidad del especialista, a largo plazo es posible obtener audición normal.

Masa cervical
Nivel de Manejo del Médico General: Diagnóstico Sospecha Tratamiento Inicial
Seguimiento Derivar

Aspectos esenciales

▪En población pediátrica la etiología más común es Congénita.

▪Origen Tumoral en la población adulta tiene mayor riesgo de ser maligna

▪La topografía de las lesiones nos entrega una orientación importante.

Caso clínico tipo

Paciente de 12 años, consulta por aumento de volumen cervical anterior izquierdo de 2


semanas, con leve eritema y aumento de calor local. Fue tratada con amoxicilina con ácido
clavulánico por 14 días. Ha presentado fiebre escasa, buen estado general, sin baja de peso.
La adenopatía es única, móvil, leve eritema, sin compromiso de otros ganglios y sin
visceromegalia. Tiene una lesión a 3 cm de la adenopatía, tipo pápula. Refiere que se la
hizo al jugar con su gato.

Definición

Aumento de volumen persistente a nivel cervical.

Etiología-epidemiología-fisiopatología

En niños y jóvenes, más del 80% de los tumores cervicales serán benignos. En los mayores
de 40 años aproximadamente el 60% serán malignos. Estas masas pueden ser de origen
congénito, inflamatorio o neoplásico.

Para su estudio es de utilidad la clasificación de Lahey que los agrupa en:

Tumores de línea media: Quiste tirogloso (tumor congénito del cuello más frecuente,
segunda causa de tumor cervical, la característica distintiva es que asciende al tragar. Se
puede inflamar por una IRA alta, el tratamiento es la resección quirúrgica de todo el
trayecto), tiroides ectópico, quiste epidermoide, teratomas, adenoma del istmo tiroideo,
linfonodo delfiano (prelaringeo).
Tumores laterales únicos: Quistes y fístulas branquiales, tumores de parótida o glándula
submaxilar, linfangioma, tumores del cuerpo carotídeo, neurilenoma, tumores vaso-
formativos, metástasis de primario desconocido, lipoma, teratoma.

Tumores laterales múltiples: Adenopatías por TBC, infecciosas, metastásicas, linfoma.

otra forma de verlos es en grupos principales de causas:

Inflamatorias: Pueden ser infecciosas (virales, bacterianas, TBC), como las adenopatías
reactivas a infecciones respiratorias o dentales, sialoadenitis, adenitis supurada, etc.
También pueden ser no infecciosas, como por ejemplo, la tiroiditis subaguda y algunas
adenopatías inflamatorias (Kawasaki, sarcoidosis, reacciones a medicamentos, etc).

Congénitas y del desarrollo: Las primeras son derivadas de restos embrionarios. Destacan
el quiste del conducto tirogloso, los quistes branquiales y el higroma quístico. Las segundas
se forman por alteraciones del desarrollo de estructuras adultas. Destacan los quistes
sebáceos y quistes dermoides.

Neoplásicas: Pueden ser tumores benignos, destacando el tumor del cuerpo carotídeo,
nódulos tiroídeos benignos, lipomas, fibromas, schwanomas, rabdomiomas, etc. También
pueden ser malignos: linfomas, adenopatías metastásicas, cáncer de tiroides, etc.

Traumáticas: por trauma directo o quirúrgico. Destacan los hematomas y los neuromas post
sección nerviosa.

La importancia relativa de estas etiologías varía según la edad a la que se presente la masa
cervical. En el grupo pediátrico (0 a 14 años), predominan las causas inflamatorias,
seguidas por las congénitas y finalmente las neoplásicas, con predominio de lesiones
malignas. En el grupo adulto joven (15 a 40 años) el orden es el mismo, pero pierden
importancia las lesiones congénitas y las lesiones tumorales benignas son más frecuentes
que las malignas. En el grupo de adultos mayores (más de 40 años), el orden cambia,
apareciendo en primer lugar las neoplasias, con predomino de tumores malignos, seguidas
de causas inflamatorias, siendo excepcionales las congénitas.

Diagnóstico

Para orientar el diagnóstico hacia la causa de la masa cervical, es necesario una anamnesis
que incluya: edad, tiempo, forma y momento de aparición de la masa, síntomas de
compromiso del estado general, síntomas de alarma tales como dolor, disfonía o disfagia.
Además de antecedentes personales medico quirúrgicos, hábitos y antecedentes familiares
de enfermedades neoplásicas.

Al examen físico: Ubicación de la masa cervical, dolor, adherencia a estructuras vecinas,


número y tamaño, signos acompañantes tales como los de patología tiroidea o adenopatías
en otras localizaciones que nos puedan orientar hacia linfoma o enfermedades infecciosas.
En base a la orientación de la clínica, se plantea la solicitud de exámenes tales como;
exámenes de laboratorio, ecografía, TAC, RM, endoscopia o nasofibroscopia, biopsia,
estudio con radioisótopos, etc.

Algunas masas cervicales tienen características especiales en su presentación clínica:

ADENOPATÍAS: SE palpan en relación a las cadenas linfáticas (submentonianas,


submandibulares, yugulares, supraclaviculares, etc.). La clínica puede orientar a algunas
etiologías, pero no es de certeza.

Adenopatías infecciosas: Suelen ser múltiples, dolorosas, móviles y asociadas a otros


síntomas, como fiebre y malestar general. Con frecuencia es evidente la causa (Ej:
faringitis, lesión dental). Los abscesos se palpan fluctuantes. Las adenopatías tuberculosas
(escrófula) pueden fistulizar a la piel.

Linfomas: suelen presentar poliadenopatías tanto cervicales, como extracervicales (axilares,


inguinales). También pueden tener síntomas sistémicos (fiebre, sudoración, baja de peso).
Se ha descrito dolor, en relación a la ingesta alcohólica en el linfoma de Hodking.

Adenopatías metastásicas: Pueden ser múltiples o única. Generalmete son indoloras, de


consistencia pétrea y adheridas a los planos. Pueden acompañarse de síntomas del tumor
primario (disfonía, disfagia). Mientras más altas son, con mayor probabilidad son
secundarias a tumores de cabeza y cuello (ej: vía aerodigestiva superior). Las más bajas
pueden originarse en estructuras extracervicales (ej: tubo digestivo).

LESIONES CONGÉNITAS: Suelen complicarse con infecciones, principalmente por


bacterias de la flora bucal.

Quiste del conducto tirogloso: es la lesión congénita más frecuente. Aparece como un
nódulo liso, indoloro en la línea media. Se detecta frecuentemente en niños de entre 2 y 10
años de edad, sin embargo puede manifestarse más tardíamente. Puede ser el único tejido
tiroídeo presente, por lo que debe evaluarse la presencia de la glándula tiroides, antes de su
resección.

Quiste branquial: Se palpan como nódulos redondeados, indoloros, de bordes lisos. Se


ubican en el borde anterior del músculo esternocleidomastoídeo. Puede tener fístulas
asociadas. Suele ser de expresión tardía, diagnosticándose comúnmente en la adolescencia.

Higroma quístico: corresponde a una malformación linfática. Suele manifestarse como una
gran masa fluctuante, ubicada preferentemente en el triángulo posterior del cuello. Suelen
aparecer antes del primer año de vida.

Quiste sebáceo y dermoides: el quiste sebáceo es la lesión del desarrollo más frecuente. Se
presenta como un quiste con un poro que retrae la piel, la eleva y se fija a ésta. Se
diferencia del quiste dermoides, en que éste se ubica profundo en relación con la fascia
cervical, por lo que la piel que lo cubre se moviliza libremente.
TUMORES: suelen ser de crecimiento lento, indoloros. Algunos presentan características
especiales.

Tumor del seno carotídeo: suele palparse en estrecha relación con la bifurcación carotídea,
como una masa pulsátil, compresible, que se rellena rápidamente al soltarla y se mueve en
sentido lateral, pero no vertical.

Nódulos tiroídeos: Se palpan en estrecha relación con la glándula tiroides. Los bocios
pueden ser difusos o nodulares (ver capítulo correspondiente). Los carcinomas suelen ser
de consistencia más dura, de bordes irregulares y se pueden asociar a adenopatías. La
tiroiditis subaguda se palpa como un bocio difuso doloroso.

dentro de los elementos que tenemos para el diagnostico destaca:

Punción aspirativa con aguja fina: Es la principal herramienta diagnóstica para un nódulo
cervical. Permite obtener muestras para citología, biopsias y cultivos. No entrega
histología. Diferencia lesiones sólidas de quísticas. No debe realizarse cuando se sospecha
un origen vascular. Junto con la ecografía es el examen de elección en el estudio del nódulo
tiroídeo. A diferencia de la biopsia quierúrgica, no altera la diseminación tumoral.

Ecografía: Es no invasiva, no irradiante, pero operador dependiente. Es insustituible en el


estudio de un bocio. Entrega gran información en el estudio de adenopatías múltiples y
patologías de las glándulas salivales. Permite realizar biopsias dirigidas. Diferencia un
flegmón de un absceso.

TAC de cuello: Es el examen radiológico de mayor utilidad en el estudio de un nódulo o


masa cervical compleja. No corresponde realizarlo ante una adenopatía única. Es de
elección en el estudio de diseminación de tumores de cabeza y cuello y en la evaluación de
abscesos y flegmones.

RNM de cuello: Tiene un rendimiento similar a la TAC. Es utilizada en tumores de la base


del cráneo o cuando nos interesa diferenciar tejido tumoral de fibrosis postquirúrgica o
postradiación. Es útil en la búsqueda de lesiones mucosas precoces en el estudio de
adenopatías metastásicas de un primario desconocido.

Cintigrafía: Su mayor utilidad es en la evaluación del nódulo tiroídeo, hipertiroídeo


(radioyodo) y en linfomas (galio).

Biopsia quirúrgica: Suele seguir a a la punción con aguja fina, sugerente de malignidad. Es
el tratamiento de algunas lesiones congénitas. Su indicación debe ser meditada, a pesar que
la mayoría de las veces termina siendo utilizada.

Tratamiento antibiótico de prueba: Ante la sospecha de una adenopatía infecciosa, se puede


tratar con antibióticos y antiinflamatorios por un máximo de 15 días. Si no responde se
debe proceder con biopsias.
Tratamiento

Dependerá de la etiología el tratamiento a realizar.

Cuando existe la sospecha clínica de una adenopatía inflamatoria y el resto del examen
físico es negativo es posible plantear tratamiento antibiótico, anti-inflamatorio y
observación por un periodo no mayor a dos semanas. Si el nódulo afectado persiste o
aumenta de tamaño, se debe continuar con el estudio diagnóstico.

Seguimiento

Salvo para las adenopatías infecciosas, el seguimiento será de responsabilidad del


especialista de acuerdo a la etiología de la masa cervical.

Odinofagias agudas
Nivel de manejo del médico general: Diagnóstico Específico Tratamiento Completo
Seguimiento Completo

Aspectos esenciales

 Se debe en su mayoría a faringoamigdalitis aguda de etiología viral.


 Bacteriana más frecuente: S. pyogenes.
 Diagnóstico clínico, pero se complementa con laboratorio.
 El tratamiento depende de la etiología.

Caso clínico tipo

Paciente de 2 años consulta por odinofagia de 3 días de evolución, asociada a coriza


abundante y fiebre hasta 37,8°C. Al examen físico faringe congestiva, sin adenopatías.

Definición

Se refiere al dolor que siente el paciente al tragar, de < 2 semanas de evolución.

Etiología-epidemiología-fisiopatología

Es una causa muy frecuente de consulta. La etiología más frecuente la constituyen los
cuadros infecciosos: desde los virales (causa más frecuente en < 3 años), bacterianas o
fúngicas hasta numerosas enfermedades sistémicas, así como también neurales y tumorales
o por cuerpo extraño. La causa más frecuente de odinofagia es la faringoamigdalitis viral,
clasificándose en 2 grupos:
 Virus que comprometen la faringe al igual que otras localizaciones del tracto
respiratorio superior, rhinovirus, coronavirus, influenza A y B y Parainfluenza.
 Virus con manifestaciones localizadas como: enterovirus (herpangina), Epstain
Barr (mononucleosis), citomegalovirus, adenovirus y virus herpes simple.

Sin embargo, entre el 15 a 30% de la faringoamigdalitis son de carácter bacteriano dentro


de las cuales, el más frecuente es el estreptococo beta hemolítico grupo A.

Diagnóstico

Anamnesis completa y examen físico. Teniendo en cuenta que la principal causa es la


faringoamigdalitis preguntar por síntomas respiratorios, fiebre, buscar adenopatías en el
examen físico. Si fue de inicio brusco y asociado a la ingesta de algún alimento pensar en
cuerpo extraño. En el diagnóstico etiológico de la faringoamigdalitis tener en cuenta sus
manifestaciones clínicas:

 Bacteriana: Más frecuente en niños mayores de 2 años. La clínica se caracteriza por


comienzo brusco, presencia de odinofagia, fiebre, cefalea, exudado confluente,
adenopatías submaxilares anteriores, náuseas, vómitos y ausencia de coriza, tos,
conjuntivitis y diarrea.
 Faringoamigdalitis viral aguda: síntomas catarrales, fiebre leve < 38°, mucosa
faríngea granular, nulo o escaso exudado amigdalino (excepto VEB, adenovirus),
úlceras y vesículas (enterovirus, herpes).
 Infección por Epstein Barr: Más frecuente en adolescentes. Existe una hipertrofia
amigdalina con exudado grisáceo capaz de ser retirado sin sangramiento,
hepatoesplenomegalia, fiebre y linfoadenopatías cervicales anteriores y posteriores.
 Orofaringitis fúngica: como el caso de la candidiasis oral, más frecuente en
lactantes que han tomado antibióticos de amplio espectro. Inicialmente: máculas
eritematosas en encías, paladar, lengua y faringe, o una fina membrana blanquecina
y raramente con adenopatías. Sospechar en diabéticos, Sd. Di George, VIH y
pacientes en quimioterapia. Como exámenes complementarios nos servirán, el
hemograma con VHS, cultivo faríngeo, test de detección para estreptococo, IgM
para EB, CMV, etc.

Tratamiento

Dependerá de la etiología de la odinofagia.

En la faringoamigdalitis el tratamiento depende de si es viral o bacteriana:

Viral: Tratamiento sintomático, con analgésicos y AINES. En general autolimitada. En caso


obstrucción de vía aérea por crecimiento amigdalino (mononucleosis
infecciosa): corticoides. En alteraciones hematológicas importantes: Ig endovenosa.

Bacteriana: Antibióticos: Penicilina Benzatina contraindicada en < de 4 años; < 25 kilos:


600.000 U IM por 1 vez; > 25 kilos: 1.200.000 U IM por 1 vez: Alternativa) Amoxicilina
75 mg/kg/día cada 8 ó 12 horas por 10 días. En caso de alergia a PNC: Eritromicina 50
mg/kg/día dividido en 4 dosis, por 10 días. Se prefiere el uso de Claritromicina 500 mg
c/12 h por 10 días. Paracetamol: 15 mg/kg/dosis, cada 8 horas.

Seguimiento

Por médico general. No requiere control. Consultar si persiste sintomatología.

Odinofagias crónicas
Nivel de manejo del médico
general: Diagnóstico Sospecha Tratamiento Inicial Seguimiento Derivar

Aspectos esenciales

 Dolor a la deglución por más de 3 meses.


 Múltiples causas.
 La clínica del cuadro orienta a la etiología.
 En algunos casos, el cambio de conducta puede ayudar en el tratamiento
inicial.

Caso clínico tipo

Paciente masculino 54 años, obeso mórbido con hábito tabáquico activo (IPA 25).
Refiere odinofagia de 1 año de evolución, motivo por el que consulta. Al
interrogatorio destaca síntomas dispepticos. Al examen físico sin hallazgos
significativos. Se educa a paciente sobre hábitos (alimenticios y vicios) y se deriva
a ORL para estudio.

Definición

Deglución dolorosa de más de 3 meses de duración.

Etiología-epidemiología-fisiopatología

Las causas de odinofagia crónica son variadas, entre las que encontramos
infecciones, tumorales, daño irritativo mecánico o por químicos, todas las cuales
tienen asociadas inflamación de la faringe. Dentro de los cuadros que pueden
presentar una odinofagia crónica encontramos:

 Inflamación, llagas, úlceras: Puede estar relacionado con trauma (por


consumo de alimentos calientes, cáusticos o después de
instrumentalización), enfermedades autoinmunes, radiación o Enfermedad
por Reflujo Gastroesofágico.
 Tumores: Cáncer de laringe, tiroides, cuello y esófago pueden causar
odinofagia. Se debe considerar como causa en pacientes de alto riesgo
como fumadores, especialmente si se acompaña de disfagia y baja de peso.
 Trastornos esofágicos: Enfermedades que afectan el esófago pueden causar
odinofagia como: Acalasia, Espasmo esofágico, úlceras esofágicas y
Divertículo de Zenker.
 Tabaquismo: El tabaco es un poderoso irritante de mucosas, pero la
odinofagia asociada se ve más frecuentemente en grandes consumidores.
 Alergias: Los alérgenos del ambiente pueden producir inflamación e
irritación de la mucosa faríngea, lo que el paciente referirá en ocasiones
como odinofagia.
 Bulimia: Se asocia al paciente que se induce el vómito de manera frecuente,
producto de la irritación por los ácidos gástricos y el daño mecánico de la
mucosa al inducirlo.

Diagnóstico

Historia clínica detallada tomando en cuenta patologías previas, características del


dolor, tiempo de evolución y síntomas asociados. Si el dolor se gatilla al tomar
bebidas o comidas calientes puede ser indicativo de lesiones en la garganata,
como úlcera. Odinofagia durante la mañana se puede relacionar con enfermedad
por reflujo gastroesofágico severa que causa faringitis por reflujo. Si la odinofagia
se presenta con disfonía o disnea puede ser indicativo de patología laríngea o
trauqueal u obstrucción parcial de la garganta.

Muchas veces se necesitan más exámenes para orientar un diagnóstico, los que
pueden incluir radiografías de cuello, TAC y laringoscopía.

Siempre descartar cáncer.

Tratamiento

Según el diagnóstico. Algunas de las etiologías pueden tratarse inicialmente con


cambios de hábitos (como el tabaquismo y pirosis). Frente a un diagnóstico poco
claro, lo mejor es referir a Otorrinolaringólogo para estudio más acabado y
tratamiento.

Seguimiento

Por especialista.
Otitis externa
Nivel de manejo por médico general:
Diagnóstico Específico. Tratamiento Completo. Seguimiento Derivar.

Aspectos esenciales

 Otitis externa difusa es la más frecuente.


 El síntoma cardinal es la otalgia.
 Diagnóstico con otoscopía.
 El tratamiento es tópico.
 Se clasifica en difusa y localizada.
 Las complicaciones incluyen: absceso periauricular, pericondritis y otitis externa
maligna.

Caso clínico tipo

Paciente de 50 años consulta por otalgia derecha, que se asocia a otorrea de mal olor e
intenso prurito. Otalgia aumenta al movilizar el pabellón auricular.

Definición

Inflamación del conducto auditivo externo.

Etiología-epidemiología-fisiopatología

Otitis externa difusa: Se desarrolla en una piel lesionada tras un procedimiento o por
modificación de las propiedades fisicoquímicas. Existen factores favorecedores como clima
cálido y húmedo, estrechez del conducto, retención de restos epidérmicos, disminución del
cerumen, dermatitis seborreica y patologías dermatolígicas, prótesis y traumatismos locales
(cotonitos, uñas, cuerpos extraños). El cerumen tiene actividad bactericida (acidos grasos,
ph cutaneo acido, pityrosporum ovale). El edema de las capas superficiales, obstrucción de
los conductos glandulares y desaparición de la capa grasa protectora genera la proliferación
de patógenos. Los gérmenes principales son la flora saprofita del CAE: Bacterias Gram (+)
y Gram (-) (Pseudomonas aeruginosa, Estafilococo, Proteus, Estreptococo, E. Coli).

Otitis externa localizada: Infección circunscrita de un folículo con formación de absceso.


Agente principal es el Staphylococcus aureus.

Otitis externa maligna: Infección grave del CAE causada de manera casi exclusiva por la
Pseudomona aeruginosa. Aparece en pacientes inmunodeprimidos, sobretodo en diabéticos
(90%). Se asocia parálisis facial. Habitual resistencia a ATB. Mortalidad elevada: 20%
cuando no hay compromiso de pares craneanos y 80% cuando la hay.

Diagnóstico
Clínica + otoscopía.

- Otitis externa difusa: Presenta otalgia de intensidad variable que aumenta al movilizar el
pabellón auricular o el trago. Otorrea, muy fluida al principio, luego purulenta, finalmente
amarillo-verdosa y de mal olor. Pueden existir adenopatías infraauriculares o mastoídeas. A
la otoscopía puede verse edema y eritema del CAE.

- Otitis externa bacteriana localizada o furúnculo: Presenta otalgia. A la otoscopía lesión


pustular, eritematosa, de un folículo piloso. Muchas veces con adenopatías asociadas.

- Otitis externa necrotizante o maligna: Erosión de la piel del conducto, como cualquier
otitis al principio. Dolor sordo, continuo, otorrea espesa. Cefalea occipital y temporal.
Tejido de granulación en el CAE. Compromiso del facial en el 75% de los casos.

Tratamiento

- Otitis eterna difusa: Corregir factores predisponentes, tratar la infección y disminuir la


inflamación. Antibióticos: de preferencia gotas con Ciprofloxacino, una alternativa es la
Neomicina. Puede ser con corticoides o sin ellos. Si es severa, usar antibióticos sistémicos
antipseudomona. Además dar AINES para el dolor. Curación bajo microscopio si es
necesario.

- Otitis externa bacteriana localizada o furúnculo: Gotas oticas, cloxacilina oral o


mupirocina. AINES, compresas tibias, y seguimiento hasta que drene. Si no sucede, hay
que realizar una incisión en la pústula (por especialista).

- Otitis externa necrotizante o maligna: Hospitalización inmediata. Antibioticoterapia local


y sistémica agresiva con aseo del CAE. Cultivo, y comenzar con ATB e.v. que cubran
Pseudomona, luego guiarse por antibiograma. Se deja tratamiento ATB por al menos 8
semanas. Derivar a especialista.

Seguimiento

Por médico general o por especialista dependiendo del tipo.

Otitis media aguda


Nivel de manejo por médico general: Diagnóstico Específico. Tratamiento Completo.
Seguimiento Completo.

Aspectos esenciales

 Condición muy prevalente en pediatría. Generalmente en el menor de 2 años.


 Síntomas cardinales: Otalgia espontánea y al movimiento de oreja, fiebre,
irritabilidad.
 Examen otoscópico imprescindible: abombamiento, opacidad y menor movilidad
timpánica.
 1/3 se origen viral.
 Agente más frecuente: Streptococcus pneumoniae. Otros: Haemophilus
influenzae y, menos frecuentemente, la Moraxella catarrhalis.
 Tratamiento inicial con amoxicilina.

Caso clínico tipo

Lactante de nueve meses se presenta con fiebre e irritabilidad que aumenta cuando se
tracciona la oreja izquierda. Membrana timpánica izquierda abombada y eritematosa, sin
movimiento con el otoscopio neumático. El resto del examen físico es normal y la niña no
tiene antecedentes de otitis.

Definición

Inflamación aguda del oído medio y trompa de Eustaquio, uni o bilateral.

Etiología-epidemiología-fisiopatología

Esta patología comienza generalmente con una infección del tracto respiratorio superior
(viral), causando edema de la mucosa y obstrucción de la trompa de Eustaquio, lo que
dificulta la ventilación y el drenaje adecuados. Se acumula líquido en el oído medio
permitiendo la proliferación de agentes infecciosos y desencadenando la OMA.

El microorganismo causal puede ingresar a la cavidad auditiva en el proceso de inflamación


y producir la infección aguda. La TE de los menores de 3 años es más corta, ancha y
horizontal, favoreciendo este mecanismo.

Suele ser una infección monobacteriana, y los gérmenes más habituales


son: Streptococcus pneumoniae 35%, H. influenzae 25%, M. catarrhalis 13%, S. pyogenes
4%.

La vía de propagación más común es la tubárica, con una diseminación retrógrada desde
infecciones del tracto respiratorio superior hacia el oído.

La mayor incidencia de la enfermedad se presenta entre los 6 meses y los 3 años de


edad debido a una peor función tubárica y a la presencia de un reservorio de bacterias
patógenas en rinofaringe por la hipertrofia adenoidea. A los 3 años más del 80% de los
niños ya han presentado un episodio. La aparición precoz de otitis media confiere un signo
de mal pronóstico.

Diagnóstico
Tímpano eritematoso y abombado. Otitis Media Aguda (OMA)

Anamnesis

 Otalgia intensa súbita.


 Fiebre.
 Irritabilidad.
 Hipoacusia

Examen físico: Puede haber otorrea serosa, serohemática o purulenta.

Al examen otoscópico: abombamiento, opacidad, eritema y disminución de la motilidad


timpánica o incluso perforación timpánica.

Tratamiento

Medidas generales: Reposo mientras dure la fiebre, calor local, aseo del pabellón con agua
hervida, no taponear el CAE.

Farmacológico: Paracetamol en caso de dolor o fiebre y Amoxicilina 75 - 100 mg/kg/día


fraccionado cada 8 - 12 hrs, por 7 días. Dentro de la segunda línea antibiótica se encuentra
la asociación amoxicilina-ácido clavulánico, eritromicina-sulfa y cefalosporinas de 2da
generación (cefuroximo). La mayoría de los niños presentan mejoría clínica dentro de 48
horas, con disminución de la fiebre y mejoría de los otros síntomas. De lo contrario, se
deben reexaminar y plantear eventualmente un cambio de antibiótico.

Se ha postulado observación diaria del niño antes de iniciar antibioticoterapia, ya que


muchos cuadros tienen etiología viral, lo cual es impracticable en Chile, por lo cual, a todo
niño con OMA se le debe tratar con antipiréticos y antibióticos. Debido a que existe poca
resistencia a betalactámicos estos siempre serán de primera línea.

El uso de gotas óticas y descongestionantes no está avalado por estudios clínicos y no se


recomienda.
Seguimiento

Si persiste fiebre y otalgia en 72 horas o hay reinicio de síntomas antes de los 7 días en un
paciente que había mejorado clínicamente, o supuración ótica por más de 3 días: enviar a
Servicio de Urgencias en sospecha de mastoiditis o meningitis.

Derivar a Otorrino frente a:

 3 o más episodios en 1 año.


 Otorrea persistente (> 15 días).
 Hipoacusia persistente (> 2 semanas).

 Otalgia intensa súbita.

 Fiebre.

 Irritabilidad.

 Hipoacusia.

Otitis media con efusión


Nivel de manejo por médico general: Diagnóstico Específico. Tratamiento Inicial.
Seguimiento Derivar.

Aspectos esenciales

 Líquido en el oído medio.


 La manifestación principal es la hipoacusia de conducción.
 No produce otalgia.
 La mayor parte remitirá espontáneamente.
 Puede llevar a alteraciones del lenguaje.

Caso clínico tipo

Consulta madre con su hijo de 5 años, porque lo tanto en el colegio como en la casa lo
encuentran muy distraído. Durante la entrevista impresiona como un déficit auditivo, y el
niño refiere sentir el oído tapado y escuchar su propia voz cuando habla. La otoscopía es
normal.

Definición

Presencia de líquido seroso o mucoso en el oído medio, sin signos de infección aguda.

Etiología-epidemiología-fisiopatología
Patología muy prevalente, presente mayoritariamente en población pediátrica,
multifactorial en que se conjugan factores anatomo-fisiológicos (disfunción de la trompa,
fisura palatina), alérgicos, infecciosos y factores propios del huésped y del medio (déficit
inmunitario, jardín infantil, exposición al humo de tabaco). La persistencia de la disfunción
tubaria provoca una presión intratimpánica negativa como resultado de la reabsorción del
aire por la mucosa de la caja. A continuación puede aparecer un transudado desde los
capilares y finalmente una metaplasia que se encargarán de secretar mayor cantidad de
líquido. Representa la causa de hipoacusia de conducción más frecuente en pacientes
pediátricos.

Diagnóstico

El diagnóstico se plantea frente a la sospecha de hipoacusia de conducción, en pacientes


pediátricos. El paciente puede referir sensación de oído tapado y autofonía, aunque
generalmente se desarrollara de forma asintomática.

El diagnóstico se realiza con un examen físico dirigido y se confirma con audiometría e


impedanciometría. La primera revela una hipoacusia de conducción de grado variable. La
impedanciometría confirma el diagnóstico al presentar un timpanograma con curva B o C y
ausencia de reflejo acústico.

Tratamiento

3 pasos a seguir:

1° Observar la evolución espontánea (esperando la mejoría).

2° Tratamiento médico (Antibióticos, Mucolíticos, Antihistamínicos, Corticoides) -->


controversial, tiene pobres resultados

3° Cirugía (tubo de ventilación transtimpánico denominado collera o diábolo (+ posible


Adenoidectomía) --> pilar del tratamiento, resolución inmediata del cuadro. La Cirugía se
plantea después de 3 meses de observación.

Seguimiento

Corresponde la derivación a especialista para el tratamiento y seguimiento.

Otitis media crónica y complicaciones


Nivel de manejo del médico general: Diagnóstico Específico Tratamiento Inicial
Seguimiento Derivar

Aspectos esenciales
 Perforación timpánica > 3 meses.
 Alta prevalencia en < 14 años en Chile.
 Grados de hipoacusia variable.
 Causa invalidez y morbimortalidad secundaria.
 El tratamiento definitivo de OMC y sus complicaciones es QUIRÚRGICO.

Caso clínico tipo

Varón de 11 años de edad consulta por presentar pérdida leve de audición del oído derecho
desde hace 6 meses. Refiere que a veces “le sale un líquido como pus” del oído
comprometido, de muy mal olor, y se ha dado cuenta que le pasa cada vez que se resfría.
No ha presentado episodios de vértigo, ni tinnitus.

Definición

Inflamación crónica de la mucosa del oído medio o de las celdas mastoídeas en presencia
de una perforación timpánica. Por consenso, se define también como perforación timpánica
mantenida más allá de 3 meses.

Etiología-epidemiología-fisiopatología

Existe alta prevalencia alta en Chile: 2-4% en menores de 14 años. Es más frecuente en
personas con bajo nivel socioeconómico, pobre higiene, tabaquismo y mala nutrición. Hay
varias causas: Alteraciones de la aireación que llevan a derrames o efusión crónicos;
defectos del desarrollo; alteraciones de la movilidad de los músculos de la tuba auditiva;
alteraciones infecciosas crónicas de adenoides y cavidades paranasales; trastornos
metabólicos, etc. Es una patología causada principalmente por Gram (-) (Pseudomona el
más importante), también S. aureus y anaerobios; en general, son infecciones
polimicrobianas. Dentro de las Otitis media crónicas existen varias formas: OMC simple
(sólo perforación), colesteatomatosa (lesiones osteíticas que van destruyendo el tejido óseo
formando pólipos y la formación de piel en el oído medio), fibroadhesiva, granulomatosa,
tuberculosa, etc.

Diagnóstico

OMC simple se caracteriza por hipoacusia de conducción, otorrea intermitente asociada a


IRA o entrada de agua al oído (mal olor), escasa otalgia (con dolor intenso, sospechar
complicación). Puede haber vértigo y/o tinnitus al comprometerse estructuras vecinas (oído
interno, sospechar laberintitis). Suele comenzar en la niñez, y presenta reagudización de
episodios de supuración en relación a infecciones respiratorias.

Examen físico: Se requiere un examen cuidadoso del oido, esto podría requerir
otomicroscopía con succión por lo que la recomendación es derivar. A la otoscopía la
membrana timpánica esta siempre comprometida (perforación que tiende a persistir),
diferentes grados de inflamación de mucosa y tímpano, y a veces lesión de la cadena
osicular. Presenta Weber lateralizado al lado lesionado (cuando es unilateral) y Rinne
negativo. El audiograma muestra grados variables de hipoacusia de transmisión o mixta.

Las complicaciones pueden ser:

Intratemporales:

 Destrucción de huesecillos.
 Parálisis facial.
 Petrositis; inflamación de la estructura trabecular del hueso temporal más allá de las
cavidades del oído medio. Poco frecuente, en menores de 7 años e
inmunodeprimidos. Produce otalgia violenta con irradiación hemicránea. TTo: ATB
más miringotomía con tubos de ventilación.
 Fístula del canal semicircular.
 LABERINTITIS; complicación más frecuente de OMC, invasión del laberinto, con
hipoacusia, alteración del equilibrio, y nistagmo. El diagnóstico se confirma con
TAC/RNM y el tratamiento es antibioterapia endovenosa y corticoides urgente.

Extratemporales

Intracraneanas:

 Meningitis (la más frecuente de OMA).


 Absceso cerebral, intradural o extradural.
 Tromboflebitis del seno lateral.
 Hidrocefalia ótica.

Extracraneales:

 Mastoiditis.
 Absceso superióstico (más frecuentes en OMA).
 Absceso de Bezold (drena al ECM).

Tratamiento

El tratamiento definitivo es quirúrgico, destinado a resolver complicaciones y prevenirlas,


resolver el cuadro inflamatorio y mejorar la audición (timpanoplastía). En el tratamiento
médico se pueden usar gotas antibióticas y antiinflamatorias de acción local, junto con
limpieza y aseo periódico con microscopio y medidas generales (impedir entrada de agua,
piscina, etc).

Seguimiento

Por especialista.
Otohematoma
Nivel de manejo del médico general: Diagnóstico Específico Tratamiento
Completo Seguimiento Derivar

Aspectos esenciales

Asociado a traumatismo del pabellón.

Se debe realizar drenaje de la colección hemática.

Puede provocar alteraciones estéticas permanentes.

Caso clínico tipo

Paciente de 24 años consulta por aumento de volumen del pabellón auricular tras
traumatismo mientras practicaba boxeo.

Definición

Colección de sangre ubicada entre el cartílago y el pericondrio del pabellón auricular.

Etiología-epidemiología-fisiopatología

Es una lesión típica de deportistas de contacto (Boxeo, lucha grecorromana, rugbi, etc.)

Se localiza fundamentalmente en la cara externa entre hélix y antihélix.

Si no se drena puede provocar pericondritis, necrosis del cartílago, con la consiguiente


deformación estética.

Diagnóstico

Antecedente de trauma en pabellón auricular. A la inspección destaca aumento de volumen


y temperatura del pabellón auricular. Revisar en detalle el pabellón buscando heridas o
picaduras. Se debe realizar otoscopía para descartar que el traumatismo haya producido
lesiones timpánicas.

Tratamiento

Se debe realizar el drenaje de la colección hemática para evitar la necrosis del cartílago y
un vendaje compresivo, con protección antibiótica para evitar una pericondritis.

Seguimiento

Por especialista.
Otorragia
Nivel de manejo del médico general: Diagnóstico Específico Tratamiento
Inicial Seguimiento Derivar

Aspectos esenciales

Con frecuencia se asocia a Trauma craneano o accidentes de tránsito.

Descartar fractura de base de cráneo.

Se debe evitar manipular y el manejo es de especialista.

Caso clínico tipo

Paciente hombre de 58 años, sin antecedentes, es traído por familiares porque tras caer
desde el techo de su casa se encuentra somnoliento, con amnesia inmediata del episodio,
otorragia e hipoacusia de oído derecho.

Definición

Hemorragia por el conducto auditivo externo (CAE).

Etiología-epidemiología-fisiopatología

Otorragia puede estar en contexto:

- Patología traumática: Fractura del peñasco, traumatismo timpánico-osicular directo

- Patología infecciosa: Otitis media aguda, Otitis media crónica, Otitis externa, miringitis
bullosa

- Patología tumoral: Paragalglioma (tumor benigno), Carcinoma epidermoide de oído


medio (tumor maligno)

Los traumatismos son la causa más frecuente de otorragia. Generalmente son las fracturas
longitudinales del hueso temporal las que producen otorragia.

También puede ser por rotura timpánica por lesión directa o barotrauma.

Diagnóstico

Anamnesis: Detallar historia clínica buscando posibles etiologías. Consignar síntomas


acompañantes tales como fiebre, prurito CAE, efusión. (OMA, OMC, otitis externa), dolor
intenso de instalación brusca seguido de otorragia puede orientarnos a perforación
traumática de la membrana timpánica. Siempre consignar si existe hipoacusia o vértigo
para evaluar compromiso de oído interno.

En caso de tratarse de traumatismos detallar el mecanismo: Manipulación inadecuada del


CAE, introducción de objetos o barotrauma, Traumatismo Encéfalo Craneano (fractura
temporal longitudinal), fractura articulación temporomandibular.

Examen físico: Depende de la causa del sangrado.

CAE: eritema, vesículas, heridas, coágulos, cuerpos extraños

Membrana timpánica: indemne, perforación, hemotímpano.

TEC: otorragia, otorraquia, otohematoma, hipoacusia de conducción unilateral, parálisis


facial, vértigo y nistagmo espontáneo. hematoma perimastoídeo (signo de Battle).

Tratamiento

Depende del diagnóstico

Evitar manipular si la indicación es derivar

Derivar de Urgencia:

Otorragia asociada a: fiebre alta no controlada, parálisis facial, vértigo, antecedentes de


patología ótica crónica (ej. OMC).

Sospecha de TEC

Sospecha de complicaciones de Otitis externa, OMA y/u OMC

Seguimiento

Por especialista.

Patología de glándulas salivales


Nivel de manejo del médico general: Diagnóstico: Inicial Tratamiento:
Inicial Seguimiento: Derivar

Aspectos esenciales

 Etiología diversa
 Gran importancia de la clínica para el diagnóstico.
 Derivar a especialista para diagnóstico específico y manejo, con indicaciones
iniciales.
 Alta sospecha de patología maligna.
 Neoplasias mucho más frecuentes en adultos

Caso clínico tipo: Mujer de 50 años consulta por aumento de volumen en el piso de la boca
de 1 semana de evolución, doloroso, de aparición súbita, que aumenta de tamaño y dolor
con las comidas. Al examen físico se observa un aumento de volumen en el lado izquierdo
del piso de la boca, con eritema local, y a la palpación bimanual se encuentra una masa
dura, inmóvil.

Definición:

Cualquier patología que comprometa glándulas salivales mayores (parótida, submandibular,


submaxilar) o menores (cerca de 1000 en la mucosa oral y faríngea

Etiología-epidemiología-fisiopatología:

Dentro de las etiologías encontramos: inflamatoria aguda (viral, bacteriana), inflamatoria


crónica no obstructiva (parotiditis recurrente, lesión linfoepitelial benigna, granulomatosa)
u obstructiva (sialolitiasis, estenosis de conducto); metabólica (sialoadenosis), traumáticas;
quistes (ránulas, mucocele); neoplasias (1-3% de tú del cuerpo) benignas (70-80% adenoma
pleomorfo; Tu de Whartin) o malignas (tú mucoepidermoide, adenoidoquístico).

Diagnóstico:

La anamnesis nos da una buena aproximación de la etiología. Consignar aumento de


volumen (duración); dolor, rubor, fiebre, trismus (inflamación?); bilateralidad, xerostomía
(sistémico?); velocidad de crecimiento (tumor?); recurrencia; relación con alimentación
(obstrucción?); función nervio facial (tú maligno?); etc. Al examen físico observar y palpar
aumento de volumen facial u orofaríngeo, examinar movilidad facial, ver características de
piel suprayacente, eritema de sitios de drenaje, características de la saliva, características de
la masa (difusa, delimitada, móvil, dolorosa). Dentro de los exámenes complementarios
usados tenemos la Rx simple, la sialografía (no usar en episodios agudos, poca utilidad en
tú), sialoTAC y sialoRNM (características de lesiones, adenopatías y metástasis), ecografía.
La angiografía y la cintigrafía son de poca utilidad. En algunos casos se debe hacer biopsia
excisional (dg y terapéutico) o PAAF.

Tratamiento:
En causas inflamatorias usar medidas generales (masaje, calor local, analgesia), ATB en
causas bacterianas y sialogogos en inflamación crónica. La causa obstructiva se trata
quirúrgicamente, al igual que quistes (marsupialización, extirpación). Las neoplasias
benignas se tratan quirúrgicamente, y las malignas según etapificación (cirugía +/-
radioterapia).

Seguimiento:

Por especialista

Perforación timpánica traumática


Nivel de manejo del médico general: Diagnóstico Específico Tratamiento
Inicial Seguimiento Derivar

Aspectos esenciales

Producto de daño directo a la membrana timpánica o a cambios de presión ambientales.

Hipoacusia como síntoma principal. Objetivar audiometría, la que además ayuda al


diagnóstico y seguimiento.

Diagnóstico específico al observar la perforación a la otoscopía.

Tratamiento médico en la mayoría de los casos.

Caso clínico tipo

Paciente masculino de 50 años, sin antecedentes médicos, consulta por cuadro de otalgia,
hipoacusia y otorrea izquierda de 3 semanas de evolución que inicia posterior a "bautizo"
de buceo autónomo recreacional. A la otoscopía se visualiza membrana timpánica
perforada.

Definición

Perforación de la membrana timpánica secundaria a una injuria mecánica o un cambio de


presión, que puede o no estar asociado a un daño de la cadena osicular y del oído interno.

Etiología-epidemiología-fisiopatología
Causas:

Lesiones iatrogénicas

Traumatismos directos de cuerpos sólidos contra la membrana

Quemaduras

Traumatismos indirectos: por cambios bruscos de presión.

Fracturas de base de cráneo.

Entre las causa más frecuentes se encuentran la iatrogenia, y el daño secundario al


inadecuado uso de hisopos en la limpieza del CAE.

Otras causas corresponden a aumentos bruscos de la presión en el medio externo (como


explosiones, disparos cercanos al oído, golpes con la palma de la mano en la oreja, entre
otros).

Determinantes de la magnitud del daño:

Intensidad y tipo de la agresión.

Estado previo de la membrana timpánica

Injurias y reparaciones previas (predispone)

Edad (a mayor edad, menos resistente)

Timpanoesclerosis (protege)

Diagnóstico

Historia clínica: Detallar mecanismos del trauma y caracterizar síntomas

Otalgia moderada a severa en relación al trauma, habitualmente no persistente.

Graduar Hipoacusia

Presencia o no de Tinitus

Vértigo: suele presentarse tras barotrauma, remitiendo espontáneamente. La presencia de


síntomas vestibulares tras un traumatismo directo debe hacer sospechar una fístula
laberíntica que exige tratamiento quirúrgico de urgencia.

Examen Físico:
Al examinar el CAE podría haber algún grado de otorragia u otorrea, habitualmente escasa.

A la otoscopía se debe objetivar la perforación, la que puede ser de morfología variable.


Más frecuente en cuadrantes posteriores e inferiores. Tendencia a aumentar de tamaño las
primeras 48 hrs por retracción.

Exámenes complementarios:

Audiometría: importante para objetivar el daño auditivo. El examen mostrará una


hipoacusia de conducción, más severa si hay compromiso de la cadena osicular. Hipoacusia
mixta debe hacer sospechar lesión de oído interno o fístula laberíntica.

TAC de peñasco:

Descartar fractura secundaria al traumatismo.

Descartar colesteatoma.

Evaluar Complicaciones:

Asociadas a la perforación:

Disyunción de la cadena osicular

Colesteatoma

Infecciones

Tratamiento

Medidas generales:

Remover material contaminante del CAE

Instruir al paciente a no introducir objetos al CAE y evitar la entrada de agua. Tomar


precauciones al bañarse.

Analgesia, de ser necesaria.

Evaluación por especialista: Alrededor de un 80% presenta cierre espontáneo durante los
primeros meses y solo requerirá manejo expectante.

Terapia antibiótica tópica (gotas) indicada solo ante signos de infección (otorrea).

Manejo quirúrgico: en caso de disyunción de la cadena osicular, o tras 6 meses sin cierre
espontáneo.
Miringoplastía (cierre de la perforación con injerto)

Timpanoplastía (cierre perforación más revisión cavidad timpánica y eventual reparación)

Evaluar complicaciones asociadas a la cirugía:

Deterioro de la audición

Reperforación

Colesteatoma

Seguimiento

Por especialista.

Poliposis nasal
Nivel de manejo del médico
general: Diagnóstico: Sospecha. Tratamiento: Inicial. Seguimiento: Derivar.

Aspectos esenciales

 Forma especial de rinosinusitis crónica


 Sospecha diagnóstica por anamnesis y examen físico: obstrucción nasal, rinorrea e hiposmia.
Diagnóstico con rinoscopía.
 Siempre completar anamnesis con: Antecedentes de asma, antecedentes de alergia a aspirina.
 Tratamiento inicial médico con corticoides tópicos, más control de atopias.

Caso clínico tipo

Paciente de sexo masculino de 25 años, refiere cuadro de alteración del olfato y voz nasalizada. Al
interrogatorio refiere cuadro intercurrentes de sinusitis (al menos 1 al año). Al examen físico sin hallazgos
significativos. Se solicita rinofibrolaringoscopia para estudio.

Definición

Lesiones benignas que crecen de la mucosa de la cavidad nasal (la gran mayoría se origina de la pared nasal
lateral y meato medio) o de los senos paranasales.

Etiología-epidemiología-fisiopatología

Es una patología bastante frecuente, tiene una prevalencia aproximada de 4% en la población general.
Usualmente se presentan en pacientes mayores de 20 años. Más frecuente en hombres.

Algunas teorías consideran los pólipos como consecuencia de condiciones que causan inflamación crónica de
la mucosa de la cavidad nasal.

La mayoría se presenta de manera bilateral. De ser unilateral es necesario hacer diagnóstico diferencial con
otras patologías tumorales. Producen obstrucción nasal y oclusión de los ostium de drenaje de los senos
paranasales y con lo que pueden ser causa de rinosinusitis aguda y crónica. Pueden modificar la forma del
hueso por compresión, remodelándolo y en ocasiones erosionándolo.

Clasificación:

1. Pólipo antrocoanal (se origina en el seno maxilar, emerge en el meato medio).


2. Pólipo grande aislado.
3. Poliposis asociada a rinosinusitis crónica no eosinofilica.
4. Poliposis asociada a rinosinusitis crónica eosinofilica.
5. Poliposis asociada a otras patologías sistémicas.
6. Patologías asociadas a poliposis: asma bronquial, fibrosis quística (importante descartar en niños),
rinitis alérgica, sinusitis fúngica alérgica, rinosinusitis crónica, triada ASA (intolerancia al ácido
acetilsalicílico, asma y poliposis nasal), disquinesia ciliar primaria, síndrome de Churg-Strauss,
síndrome de Young (sinusitis crónica, poliposis nasal, azoospermia), NARES (nonallergic rinitis with
eosinophilia syndrome).

Diagnóstico

Clínica: Obstrucción nasal que se expresa como:

 Dificultad para respirar por la nariz: respiración bucal, ronquidos.


 Alteración del olfato (hiposmia, anosmia).
 Voz nasalizada.
 Otros síntomas: rinorrea frecuente o abundante, prurito nasal, cefalea, algia facial, descarga
posterior Poliposis masiva (raro; pacientes con fibrosis quística o sinusitis fúngica alérgica) puede
producir alteraciones de las estructuras craneofaciales: proptosis, hipertelorismo, diplopía.

Examen Físico:

 Especuloscopía nasal.
 Rinofibrolaringoscopia: observar los pólipos directamente; presencia de secreción mucopurulenta
debe hacer sospechar una sobreinfección.

Otros Exámenes:

 Un TAC de cavidades paranasales permitirá precisar la dimensión de los pólipos y evaluar los senos
paranasales.
 Estudio de procesos alérgicos asociados.
 Estudio del exudado nasal (buscar eosinofilia - NARES).
 Cultivo de secreciones nasales: identificar patógeno responsable de una sobreinfección.

Tratamiento

Tratamiento Inicial:

Corticoterapia

1. Tópicos: de elección. Reduce el tamaño de los pólipos y previene recurrencias.


2. Sistémicos: no de rutina por efectos adversos. Mayor utilidad previo a cirugía.

Antihistamínicos, en contexto de patología alérgica.

Derivar a Especialista: en caso de poliposis severa refractaria a tratamiento médico que requiera manejo
quirúrgico.
Seguimiento

Por especialista

Rinorrea crónica
Nivel de manejo del médico
general: Diagnóstico: Sospecha, Tratamiento: Inicial, Seguimiento: Derivar

Aspectos esenciales

 La gran mayoría será un signo de una patología concomitante.


 Sospecha diagnóstica principalmente por anamnesis y examen físico.
 Realizar búsqueda minuciosa de posibles causas y factores que contribuyan al
cuadro.
 Tratamiento médico sintomático y derivación para patología específica.
 Seguimiento por especialista.

Caso clínico tipo

Paciente femenino 15 años, traída por madre debido a cuadro de secreción nasal abundante
de larga data, que se intensifica durante la primavera. A la anamnesis destaca antecedente
de madre asmática y hermana con dermatitis alérgica. Se deriva a ORL para estudio.

Definición

Secreción nasal constante y abundante por periodos de tiempo prolongado, generalmente


secundario a otra patología subyacente.

Etiología-epidemiología-fisiopatología

Causas más frecuentes:

 Rinitis alérgica.
 Rinosinusitis
 Secundaria a obstrucción nasal posterior.
 Hipertrofia de adenoides.
 Cuerpo extraño.
 Rinitis crónica atrófica (OCENA): inflamación crónica de la mucosa nasal, más
sequedad por atrofia de la misma. En la clínica se ven fosas nasales amplias,
rinorrea crónica y fétidas costras intranasales adherentes, que si se sacan se produce
hemorragia. Se caracteriza por el mal olor.
 Sobre uso de descongestionante nasal.

Tipos de rinorrea: serosa, mucosa, purulenta, hemática, o asociaciones de ellas.

Factores que favorecen el desarrollo de una rinorrea crónica:

 Factores obstructivos: cuerpo extraño nasal, hipertrofia de adenoides (en niños),


desviaciones severas del septum nasal, obstrucciones del complejo osteomeatal
(propicia una sobreinfección sinusal), tumores y pólipos nasales, hipertrofia de
cornetes inferiores, estenosis de coanas.
 Aumento de la producción de moco nasal: principalmente debido a rinitis alérgicas,
infecciones, y rinitis no alérgicas.
 Infecciones: localizadas o por contigüidad ( por ejemplo, cavidades paranasales ),
principalmente bacterianas: s. pneumoniae y h. influenzae. Con menos frecuencia se
ven infecciones por hongos.
 Alteraciones en el transporte mucociliar: por disquinesia ciliar, o secundario a un
aumento en la cantidad y composición del moco nasal.
 Inmunodeficiencias: de tipo humoral, y por déficit de producción de
inmunoglobulinas IgA e IgG (y de subclases de IgG).

Diagnóstico

En la anamnesis próxima, determinar las características de la rinorrea (acuoso, mucoso,


purulento, sanguinolento), preguntar dirigidamente por estacionalidad, ubicación del
paciente y potenciales gatillantes (buscando causa alérgica). En la anamnesis
remota indagar antecedentes del paciente (DM, inmunocompromiso), patología ORL previa
y antecedentes familiares. El diagnóstico de una rinorrea crónica por sí solo es, en la gran
mayoría de los casos, insuficiente para enfocar exitosamente un tratamiento. Por esto, al
momento del diagnóstico debe realizarse un examen otorrinolaringológico completo para
encontrar una posible causa del cuadro descrito, acompañarlo de la instrumentalización más
certera posible (ej: nasofibroscopia), y de los exámenes imagenológicos que el médico
considere pertinentes al caso.

Otros Exámenes:

 Radiografías de cavum y de cavidades paranasales (baja sensibilidad y


especificidad).
 TAC de cavidades paranasales.
 Exámenes de alergia (IgE, prick test).
 Estudio de eosinofilia en secreción nasal.
 Test del sudor, estudio inmunológico.
Tratamiento

Inicial:

 Tratamiento tópico sintomático de la rinorrea.


 Medidas ambientales generales. Evitar alérgenos.
 Aseo nasal frecuente, con solución salina.
 Descongestionantes tópicos: Oximetazolina, disminuye la hipersecreción.
 Antihistamínicos: rinitis alérgicas, mejora el prurito nasal, estornudos; también
contribuyen a disminuir la vasodilatación local y con esto, la hipersecreción e
hidrorrea.
 Corticoides: si se evidencia obstrucción. Sistémicos o tópicos.

Específico: derivar a especialista.

Seguimiento: Por especialista

Rinorreas agudas
Nivel de manejo del médico
general: Diagnóstico: Específico, Tratamiento: Completo, Seguimiento: Completo

Aspectos esenciales

 Secreción nasal de < de 4 semanas de evolución.


 La causa más frecuente es el resfrío común.
 La complicación más importante en el lactante es la otitis media aguda y en el
escolar la sinusitis.
 El tratamiento depende de la patología.

Caso clínico tipo

Paciente de 4 años consulta por cuadro de 3 días de evolución de rinorrea serosa abundante
y sensación de obstrucción nasal, asociada a estornudos frecuentes y fiebre en 2 episodios
37,8°C. Al examen físico faringe congestiva y coriza. Sin alteraciones al examen pulmonar.

Definición

Secreción nasal de < 4 semanas de evolución.


Etiología-epidemiología-fisiopatología

Las causas más comunes son los cuadros virales y las reacciones alérgicas. El carácter de la
rinorrea puede orientar a veces a su origen. En caso de una rinorrea de
aspecto purulento puede presentarse tanto en cuadros virales con sobreinfección bacteriana.
La presencia de sangre o la secreción unilateral obliga a descartar siempre un tumor,
especialmente en adultos. Características como la uni o bilateralidad de la secreción
también pueden orientar, por ejemplo, en rinorrea purulenta unilateral en un niño, siempre
descartar un cuerpo extraño.

Resfrío común: su etiología es predominantemente viral. Los agentes más importantes son
los rinovirus, (más de 100 serotipos), coronavirus y el VRS. Los niños presentan en
promedio 5 a 8 cuadros al año, con incidencia máxima en el menor de dos años. Esta
frecuencia se mantiene alta a lo largo de la vida, con cuadros más leves, con promedio de 2
a 4 resfríos al año en el adulto. El período de incubación es corto, incluso de pocas horas, y
el cuadro dura generalmente de 3 a 7 días.

Rinosinusitis aguda: resfrío común que dura más de 7 a 10 días o exacerbación de síntomas
luego de 5 días del curso de un resfrío común. La fisiopatología consiste en obstrucción del
ostium edema de la mucosa sinusal presión negativa dentro de la cavidad sinusal
disminución de la PO2alteración de la función ciliaracumulación de secrecionesinfección
bacteriana secundaria. Los agentes más frecuentes son Neumococo y Haemophilus
influenzae.

Rinitis vasomotora: frecuente en adultos y adulto mayor. Se produce un desajuste entre el


sistema parasimpático / simpático. Tiene test cutáneo (-). Se produce rinorrea
desencadenada por gatillantes térmicos y alimentos.

Cuerpo extraño: El cuerpo extraño en la fosa nasal provoca primero síntomas y signos de
irritación nasal unilateral, posteriormente se infecta, convirtiendo la rinorrea seromucosa en
purulenta y de mal olor (cacosmia objetiva). Más frecuente en niños.

Alergia: Los síntomas son estacionales o se exacerban con posibles gatillantes. Secreción
acuosa, estornudos, picazón de ojos.

Mucormicosis: infección de la cavidad nasal y paranasal por hongos del orden mucoral,
más frecuente en inmunodeprimidos (DM).

Diagnóstico

Iniciar con la historia del cuadro actual, determinar las características de la rinorrea
(acuoso, mucoso, purulento, sanguinolento), si es uni o bilateral, luego determinar si es un
episodio aislado o presenta recurencias. En el caso de ser recurrente preguntar por
estacionalidad y exposición a posibles gatillantes. Preguntar en anamnesis remota si tiene
antecedentes de alergias conocidas, diabetes mellitus o inmunocompromiso. Preguntar
dirigidamente por uso de descongestionantes nasales (Por rinorrea por sobreuso de
descongestionantes). Tener presente algunos signos de alarma como descarga unilateral, en
especial si es purulento o sanguinolento o algia facial.

A continuación se describe la forma de presentación de las etiologías más comunes:

Resfrío Común: El diagnóstico es clínico: fundamentalmente rinorrea y obstrucción nasal.


En los lactantes el cuadro comienza habitualmente con fiebre (no se extiende más de 72
horas), irritabilidad, decaimiento, estornudos y ruidos nasales. Pronto aparece rinorrea,
inicialmente serosa, que se va transformando en mucosa al pasar los días hasta adquirir
aspecto mucopurulento y desaparecer dentro de la primera semana. Mientras más pequeño
el niño, más depende de su respiración nasal, por lo que esta obstrucción puede incluso
producir síntomas de dificultad respiratoria. Al examen físico sólo se objetiva congestión
faríngea y presencia de coriza. Los síntomas comienzan a disminuir hacia el cuarto día. A
mayor edad, el cuadro comienza con sensación de sequedad e irritación nasal, seguido de
estornudos y coriza serosa. Otros síntomas como mialgias, cefalea, fiebre baja y tos pueden
estar presentes.

Rinosinusitis aguda: Criterios mayores: Presión / dolor facial; Congestión / plenitud facial;
Obstrucción / bloqueo nasal; Rinorrea / descarga posterior purulenta; Hiposmia /anosmia;
Pus meato medio; Fiebre. Criterios menores: Cefalea; Halitosis; Fatiga; Dolor dentario;
Tos; Abombamiento / presión / dolor ótico. Diagnóstico clínico. A menudo se presenta
como resfrío que se prolonga o empeora.

Mucormicosis: la sospecha clínica se hace en pacientes con antecedentes de


inmunocompromiso (DM, SIDA,etc), ronorrea mucopurulenta sanguínea, diplopía,
poptosis, cefalea, fiebre, eritema de la zona de los senos paranasales, dolor sinusal. Se
confirma con biopsia. El estudio se hace con TAC/RNM.

Tratamiento:

Tratamiento según etiología.

Resfrío común: Principalmente sintomático, con reposo relativo dependiendo de la edad,


adecuada hidratación y uso de antipiréticos en caso de fiebre. En los lactantes más
pequeños es fundamental realizar un buen aseo nasal en forma frecuente. El uso de
antihistamínicos y vasoconstrictores es discutido. El uso profiláctico de antibióticos está
completamente contraindicado.

Rinosinusitis aguda: Tratamiento antibiótico (Amoxicilina por 7


días); Descongestionantes tópicos, Corticoides tópicos.

Mucormicosis: quirúrgico (lo más importante) y antimicóticos ev. Derivar.


Seguimiento:

Derivar en caso de sinusitis rebelde a tratamiento, sospecha de tumor de CPN, Sospecha de


sinusitis complicada (Orbitaria, SNC), Imagen radiológica dudosa.

Síndrome de apnea del sueño


Nivel de manejo del médico general:
Diagnóstico Sospecha, Tratamiento Inicial, Seguimiento Derivar.

Aspectos esenciales

 Patología asociada a cierto grado de deterioro funcional.


 Sospecha clínica por anamnesis y presencia de factores de riesgo.
 Es importante el manejo de factores que contribuyan a la obstrucción, como
fármacos, postura, hábitos, entre otros.
 El tratamiento de patología específica de vía aérea y el seguimiento es por
especialista.

Caso clínico tipo

Paciente masculino 55 años, obeso y fumador. Refiere hipersomnolencia diurna,


irritabilidad y baja del rendimiento en el trabajo. Evaluado por psicólogo descarta trastorno
del ánimo. Al interrogatorio dirigido destaca roncopatía de larga data.

Definición

Episodios recurrentes de colapsos parciales (hipoapneas) o totales de la vía aérea superior


durante el sueño, en que ocurre un cese del flujo de aire por 10 segundos o más, pese a
persistir los esfuerzos ventilatorios (esto diferencia una apnea obstructiva de una central).
La caída de la saturación arterial de oxígeno conduce a un breve despertar característico del
cuadro.

Etiología-epidemiología-fisiopatología

El colapso de la vía puede producirse por reducción del tono muscular orofaríngeo y/o
palatino, o por estrechez del pasaje de la vía aérea desde la rinofaringe a la base de la
lengua. Los microdespertares permiten normalizar el tono muscular o corregir la postura
para desobstruir la vía aérea. Factores de riesgo:
 Anatómicos: Hipertrofia amigdalina (también adenoidea en niños), macroglosia,
hipertrofia cornetes nasales, pólipos o tumores nasales, desviación del septum nasal,
obesidad cervical, elongación del velo del paladar y de la úvula, micrognatia y
retrognatia.
 Posicional: más frecuente en decúbito dorsal.
 Edad: Riesgo aumenta con la edad.
 Obesidad.
 Condiciones médicas: Hay condiciones médicas que aumentan el riesgo como el
embarazo, falla cardíaca congestiva, enfermedad pulmonar crónica e
hipotiroidismo.
 Otros: ingesta alimentaria abundante previo a dormir, alcohol, drogas
tranquilizantes (hipnóticos, relajantes musculares).

La patología suele asociarse a un deterioro funcional durante el día debido a la interrupción


constante de las fases del sueño debido a los despertares, con mayor probabilidad de
accidentes de tránsito, laborales, trastornos del ánimo, trastornos conductuales y síndromes
cardiovasculares.

Diagnóstico

Anamnesis:

 Historial de ronquidos durante el sueño (referido por terceros).


 Presencia de factores de riesgo.
 Síntomas de cansancio durante el día.

Examen Físico:

 Examen de cabeza, cuello, cavidad nasal, cavidad oral, naso y orofaringe, buscando
posibles puntos de obstrucción que refuercen la sospecha diagnóstica.
 En caso de disponer de nasofibroscopía, realizar examen de laringe y cuerdas
vocales, con el mismo objetivo.

Polisomnografía: Otorga el diagnóstico de certeza una vez hecha la sospecha diagnóstica


con la historia clínica. Incluye un registro EEG, electromiográfico, electrooculográfico,
ECG, saturación de oxígeno, y medición de flujo nasal y bucal.

Según el número de apneas e hipoapneas o “eventos respiratorios” por hora (lo normal es
hasta 5), se clasifican en:

 Apneas leves: más de 5 y hasta 20 eventos por hora.


 Apneas moderadas: más de 20 y hasta 50 eventos por hora.
 Apneas severas: sobre 50 eventos.

Según el grado de desaturación de oxígeno que se alcance:


 Desaturación leve: hasta 85%
 Desaturación moderada: hasta 75%
 Desaturación severa: bajo 75%

Número de microdespertares por hora de sueño (lo normal es hasta 10). Lo relevante es
establecer una relación temporal con los periodos de apnea.

Tratamiento

Apneas Leves:

 Medidas Generales: alimentación, hábitos (OH, drogas), controlar fármacos,


posición al dormir y bajar de peso.
 Manejo de patología sinusal: Derivar Especialista.

Apneas moderadas a severas:

 Uso de CPAP y de BIPAP (Introducen aire (ambiental) a presión en la vía aérea del
paciente a través de una mascarilla.
 Manejo Quirúrgico: en casos particulares, con un profundo estudio en cada caso;
dirigido al punto de colapso y/o obstrucción de la vía aérea.

Seguimiento

Por especialista

Síndrome respirador bucal


Nivel de manejo del médico general:
Diagnóstico Específico, Tratamiento Inicial, Seguimiento Completo.

Aspectos esenciales

 Múltiples causas.
 Las causas más frecuentes son Hiperplasia adenoidea y/o amigdalina.
 El diagnóstico es clínico, se debe buscar etiología.
 El tratamiento precoz puede evitar las repercusiones.

Caso clínico tipo

Paciente de 6 años consulta debido a historia de larga data de roncopatía asociada a pausas
respiratorias, tos nocturna y dificultad respiratoria. Durante el último año ha desarrollado 4
episodios de amigdalitis aguda. Al examen físico se observa hiperplasia amigdalina grado
IV.
Definición

Entidad clínica que ocurre en los niños cuando respiran por la boca perenne, ocasional o
intermitentemente, ya sea por causas obstructivas, hábito o por anatomía.

Etiología-epidemiología-fisiopatología

La respiración bucal, propicia un terreno para que exista un aumento de las secreciones en
las vías aéreas superiores y que a su vez favorecerá la alteración de la funcionalidad de los
oídos, senos paranasales y con frecuencia desencadena crisis obstructivas bronquiales a
repetición.

Las causas se pueden clasificar en: a) orgánicas, provocado por un obstáculo mecánico en
nariz, faringe o boca; b) funcionales, que son quienes se beneficiarían de una resolución
quirúrgica y c) las causadas por problemas neurológicos. Las causas más comunes que
provocan respiración bucal son: hiperplasia adenoidea y/o amigdalina, rinitis; alergias,
desviación del tabique nasal, hábito de succión prolongado del pulgar (más allá del primer a
segundo año de vida), mal oclusión dentaria, enfermedades neuromusculares o retraso
psicomotor; características del desarrollo craneofacial de incidencia heredo-familiar.

Diagnóstico

Síntomas y signos más frecuentes del respirador bucal: tos seca, tos nocturna, dificultad
respiratoria, apneas obstructivas del sueño, trastornos en el desarrollo del macizo facial
(paladar estrecho y hundido, mandíbula elongada), deglución atípica, dientes mal
implantados. Como consecuencia pueden desencadenar: sinusitis, otitis, trastornos de la
audición, falta de atención, retraso escolar, alteraciones posturales.

Repercusiones Locales: Fascie adenoidea: labio inferior hipotónico (interpuesto entre los
dientes), labio superior hipertónico (retraído, corto); boca entreabierta (babeo);
pigmentación suborbital (ojeras); lengua baja proyectada hacia adelante.

Repercusiones Generales:

 Respiratorias: deformidad torácica, síndrome de apnea del sueño y muerte súbita del
lactante.
 Cardiovascular: hipertensión pulmonar, insuficiencia cardiaca derecha.
 Sistema nervioso: hipersomnia diurna, cefalea matinal, bajo rendimiento escolar,
irritabilidad y agresividad.
 Sistema gastrointestinal: dificultad para alimentarse y aerofagia.

Tratamiento

Multidisciplinario. El médico general debe realizar la derivación según la causa


sospechada.
Intervención del otorrinolaringólogo: tiene como objetivo despejar las vías aéreas
superiores obstruidas que causan la respiración bucal: adenoides, hipertrofia de amígdalas,
hipertrofia de cornetes, desviación del tabique nasal, pólipos (engrosamiento de la mucosa
nasal y sinusal) y alergias.

Criterios de amigdalectomía:

 Hiperplasia severa (grados III – IV).


 Apnea obstructiva del sueño.
 Amigdalitis recurrentes (7 episodios en 1 año, 5 en 2 años o 3 en 3 años de
observación).
 Sospecha de cáncer.
 Absceso periamigdalino (segundo episodio).
 Tonsilolitiasis (relativa, considerar calidad de vida).

Criterios de adenoidectomía:

 Apnea obstructiva del sueño;


 Hiperplasia severa que cause obstrucción respiratoria;
 Adenoiditis recurrente;
 Otitis media con efusión;
 Rinosinusitis recurrente. Solicitar “radiografía de cavum” para determinar grado de
severidad. Radiografía sirve para complementar diagnóstico, pero decisión
quirúrgica se efectúa con la clínica.

Seguimiento

Por especialista.

Síndrome vertiginoso
Nivel de manejo del médico general: Diagnóstico Sospecha Tratamiento Inicial
Seguimiento Derivar

Aspectos esenciales

 Se debe diferenciar si es periférico o central.


 SVP suele ser transitorio y de curso benigno.
 Confirmación diagnóstica la realiza el médico especialista.
 Tratamiento es según la etiología del cuadro.

Caso clínico tipo


Paciente mujer, de 50 años consulta por presentar, desde hace dos semanas, sensación de
rotación de los objetos que la rodean (de 25 segundos aprox.) todo esto debido a ciertos
movimientos de extensión, flexión y rotación de la cabeza. Tales vértigos se repiten varias
veces al día y se han acompañado de náuseas.

Definición

Percepción rotatoria del cuerpo (vértigo subjetivo) o del ambiente (vértigo objetivo), que
puede ser horizontal o vertical Diferenciarlo del mareo, que es una sensación subjetiva de
inestabilidad.

Etiología-epidemiología-fisiopatología

El vértigo es un motivo de consulta frecuente, llegando a constituir un 3-5% de las


atenciones médicas en adultos. La mayoría (sobre 90%) de los síndromes vestibulares es de
origen periférico (SVP), es decir, el sitio de la lesión causal se encuentra en canales
semicirculares (CSC), utrículo, sáculo o nervio vestibular en su trayecto dentro del hueso
temporal. Son generalmente de curso benigno y es relativamente frecuente la presencia de
síntomas cocleares. Por otro lado, los SV centrales (SVC) son menos frecuentes (5-10%) y
se producen por alguna alteración en tronco encefálico, ángulo pontocerebeloso, cerebelo o,
excepcionalmente, en parénquima cerebral o IV ventrículo. Síndromes vestibulares:
Periféricos: Neuritis vestibular. Hidrops endolinfático. Vértigo paroxístico postural
benigno. Centrales. Síndrome del ángulo pontocerebeloso. Síndrome de línea media fosa
posterior. Síndrome cerebeloso.

Periférico Central
Inicio Repentino Lento
Mal definido, menos
Gravedad del vértigo Rotación intensa
intenso
Patrón Paroxístico, intermitente Constante
Agravado por
Sí No
postura/movimiento
Náusea/diaforesis Frecuentes Infrecuentes
Nistagmo Rotatorio/Horizontal/Vertical Vertical
Fatiga de síntomas Sí No
Pérdida auditiva Puede ocurrir No ocurre
Membrana del tímpano anormal Puede ocurrir No ocurre
Síntomas/signos SNC Ausentes Por lo general, sí

Diagnóstico

Anamnesis

 Duración de la crisis (segundos, orienta más a vértigo postural, si dura horas puede
ser por alteraciones de la endolinfa).
 Forma de presentación: habitualmente de tipo episódico.
 Factores desencadenantes (espontáneo, movimientos de cabeza).
 Síntomas auditivos: hipoacusias, tinnitus.
 Síntomas vegetativos: náuseas, vómitos.
 Fármacos: ototóxicos (aminoglucósidos).
 Antecedentes familiares: migraña, convulsiones, epilepsia.

Examen Físico:

 Evaluación de la marcha
 Prueba de Romberg (evalúa alteraciones de la vía vestibular o de la propiocepción).
 Examen neurológico: completo, con énfasis en compromiso de pares craneales VII
y VIII, nistagmo, síndrome cerebeloso, examen vestibular.
 Examen ORL: evaluación de la audición y estructuras del oído. Prueba de
provocación canal semicircular posterior (CSP) .

Pruebas complementarias:

 Pruebas auditivas: audiometría


 Octavo par.
 Imágenes: RM cerebral y TC cerebral (examen de elección para diagnosticar
alteraciones de la parte ósea del hueso temporal).

Diagnóstico Diferencial

Neuronitis Vestibular: inflamación viral a nivel del ganglio de Scarpa de un nervio


vestibular. Se ve preferentemente en adultos.

Cuadro clínico: inicio súbito de vértigo intenso y persistente, acompañado de náuseas y


vómitos e inestabilidad en la marcha. Sin tinnitus ni hipoacusia. Son crisis de días o
semanas de duración, que se presentan esporádicamente en un lapso de 2 meses hasta 2
años desde el inicio de los síntomas. Examen: en el VIII par se ve paresia vestibular
unilateral en prueba calórica.

Vértigo postural paroxístico Benigno: causado por desprendimiento de otolitos. Clínica:


crisis cortas (de minutos), poco intensas, de vértigo, asociado a cambios de posición
cefálica. Son fatigables, con nistagmo postural característico de lesiones periféricas.
Examen del VIII par sin alteraciones:

Enfermedad de Menière: hidropesía endolinfática recurrencial de causa desconocida.


Clínica: crisis de vértigo de horas a días de duración, con náuseas, vómitos, síntomas
neurovegetativos, y precedidas por días de sensación de oído abombado, hipoacusia y
tinnitus.

Tratamiento
Dependerá de la etiología del cuadro y de la forma de presentación de este. Como manejo
inicial indicar reposo y antiheméticos. Si no hay defectos neurológicos o metabólicos que
se puedan identificar, el reposo durante dos o tres días es lo más eficaz como el inicio de
tratamiento en la mayoría de los casos.

Hidropesía endolinfática: En la fase aguda sedantes y antiheméticos. En los intervalos se


benedician de diuréticos y régimen hiposódico.

Neuritis vestibular: En las crisis indicar hidratación, antiheméticos y sedación. No dar


antivirales.

Vértigo paroxístico postural benigno: Maniobras de reposición. Semont y Epley.

Laberintitis: suele requerir ingreso hospitalario para asociar al tratamiento sintomático,


antibiótico y corticoide intravenosos.

Seguimiento

Derivar.

Síndrome vertiginoso agudo


Nivel de manejeo del médico
general: Diagnóstico: Sospecha. Tratamiento: Inicial. Seguimiento: Derivar.

Aspectos esenciales

Diagnóstico clínico.

Por lo general, inicio repentino orienta a vértigo periférico.

En caso de cefalea intensa, aparición brusca con factores de riesgo, y un cuadro que se escapa de los
cuadros típicos de vértigo periférico se recomienda imágenes (RNM) para descartar ACV.

Caso clínico tipo

Paciente se sexo femenino de 40 años consulta por presentar en el último día 3 episodios de segundos de
duración caracterizados por vértigo que aparece ante cambios de posición. Niega otras alteraciones de
importancia como tinnitus o hipoacusia.

Definición

Percepción rotatoria del cuerpo o del ambiente de instalación aguda.

Etiología-epidemiología-fisiopatología

Según la forma de presentación puede obedecer a múltiples causas. De acuerdo a su origen se pueden
dividir en centrales o periféricos. Lo más frecuente es el vértigo de origen periférico, y el hecho de que sea
de instalación aguda orienta aún más a esta causa.
Periférico Central
Inicio Repentino Lento
Gravedad del vértigo Rotación intensa Mal definido, menos intenso
Patrón Paroxístico, intermitente Constante
Agravado por postura/movimiento Sí No
Náusea/diaforesis Frecuentes Infrecuentes
Nistagmo Rotatorio/Horizontal/Vertical Vertical
Fatiga de síntomas Sí No
Pérdida auditiva Puede ocurrir No ocurre
Membrana del tímpano anormal Puede ocurrir No ocurre
Síntomas/signos SNC Ausentes Por lo general, sí

Diagnóstico

Detallar en la anamnesis el modo de inicio y tiempo de duración, además identificar desencadenante


(espontáneo, movimientos de cabeza) o si presenta otros síntomas asociados como hipoacusia, tinnitus o
cefalea.

Según el tiempo de duración de los síntomas, los podemos agrupar en:

- Segundos a minutos: Si se asocia a cambios posicionales corporales o de la cabeza, con una corta duración
se piensa en vértigo postural paroxístico benigno (VPPB).

- Minutos a horas: Si se acompaña de tinnitus, hipoacusia y sensación de plenitud en el oído previa al


vértigo, nos orienta a Sd. de Meniere. Si es más bien brusco, sin estos síntomas previos y asociado a
factores de riesgo (HTA, enf vascular, DM, etc) se puede tratar de un TIA del territorio vertebro basilar.

- Días (incluso semanas): Inicio lento, que tiene su peak al primer día y luego va disminuyendo en los días
siguientes es sugerente de neuritis vestibular (autoinmune, viral) . El vértigo puede demorar hasta una
semana en desaparecer y varias semanas hasta recuperar la normalidad. Puede reaparecer en un período
variable de tiempo.

Si es de aparición brusca, con factores de riesgo vasculares, puede tratarse de un AVE, que si es de tronco
se asociará a disfagia, diplopia, disartria u otro déficit focal.

Tratamiento

Dependerá de la alteración de base. En el tratamiento inicial indicar reposo y antivertiginosos. Derivar de


urgencia si se sospecha accidente vascular.

Tratamiento de las causas más frecuentes:

 Vértigo Postural Paroxístico Benigno: Informar naturaleza autolimitada del cuadro, derivar
electivamente para tto definitivo (maniobras de reposición, Semont y Epley).
 Sd. de Meniere: Reposo, antivertiginosos y antieméticos (idealmente parenterales). En el manejo
intecrisis se recomienda restricción de sal y diuréticos. Derivar a especialista para eventual
resolución quirúrgica.
 Neuritis vestibular: Durante las crisis, reposo, antivertiginosos y antieméticos. Posterior a la crisis
ejercicios + una rápida incorporación a la vida activa para facilitar recuperación.

Seguimiento

Derivar.
Sinusopatías crónicas
Nivel de manejo del médico general: Diagnóstico Específico Tratamiento Inicial
Seguimiento Derivar

Aspectos esenciales

 Rinosinusitis que perdura por al menos 12 semanas consecutivas.


 Siempre requiere estudio (TAC EN TODOS)- La radiografía de cavidades
paranasales NO sirve.
 Recordar la poliposis como una forma de Rinosinusitis Crónica.
 El tratamiento es con antibióticos de segunda línea (prolongado).

Caso clínico tipo

Paciente de sexo masculino de 19 años consulta por cuadro de rinorrea purulenta, presión
facial e hiposmia de 3 meses de evolución. El paciente refiere como antecedente alergia al
polen. ¿Que sospecha y que examen solicita?.

 Rinitis alérgica – Prick test


 RSC – RX de cavidades paranasales
 Sinusitis aguda – TAC
 RSC – TAC
 RSA – Endoscopia

Definición

Inflamación de la mucosa de la nariz y cavidades paranasales durante al menos 12 semanas


consecutivas.

Etiología-epidemiología-fisiopatología

En la rinosinusitis crónica (RSC), a diferencia de la aguda, predomina la inflamación antes


que la infección. Existen factores asociados a la RSC, los que hay que buscar y tratar:
Factores sistémicos, huésped: Inmunodeficiencias, alergias, genéticos, disfunción
mucociliar, endocrinos. Factores locales: Neoplasias, anatómicos (Hipertrofia cornetes,
septodesvicación, pólipos). Factores ambientales: Noxas ambiente, microorganismos,
trauma, cirugía, medicamentos. Se clasifica desde un punto de vista clínica en RSC con
pólipos, sin pólipos y rinosinusitis fúngica.

Diagnóstico

Los criterios clínicos diagnósticos de RSC son los mismos que los de sinusitis aguda,
excepto que en general, no presentan fiebre (mayores: presión facial, plenitud facial,
obstrucción nasal, rinorrea purulenta, pus en el meato medio, hiposmia – menores: cefalea,
halitosis, fatiga, odontalgia, febrícula, tos, otalgia); los que deben persistir al menos durante
12 semanas consecutivas. Los síntomas son por lo general de menor severidad que en la
rinosinusitis aguda.

En todos los pacientes se debe realizar estudio de imagen de cavidades paranasales con
TAC (considerar RNM cuando se sospechan complicaciones o se debe confirmar
compromiso fúngico) complementado con endoscopia nasal (que visualizará pólipos si
están presentes). Los pólipos son una forma especial de RSC caracterizada por formaciones
benignas que crecen en especial a partir de la mucosa etmoidal, sobre todo alrededor del
cornete medio. El cultivo y antibiograma de secreciones paranasales es aconsejable.

Tratamiento

Manejo por ORL. Considerar antibióticos de “segunda línea” (amoxicilina –ácido


clavulánico; levofloxacino, moxifloxacino; cefuroxima; clindamicina) durante 4 – 12
semanas; asociado a descongestionantes (antihistamínicos, corticoides inhalados u orales).
La cirugía se indica en: RSC rebelde a tratamiento, poliposis masiva o rebelde a
tratamiento, RSFA en el adulto, complicaciones, sospecha de tumor de CPN.

Seguimiento

Por especialista. Idealmente controlar al mes de tratamiento, si mejoran síntomas realizar


precozmente nueva TAC (2 semanas después) y suspender terapia si existe regresión total
(repitiendo posteriormente TAC). En caso contrario continuar el mismo tratamiento o
considerar cambiar el antimicrobiano dependiendo del cultivo.

Sordera súbita
Nivel de manejo del médico
general: Diagnóstico Específico Tratamiento Inicial Seguimiento Derivar

Aspectos esenciales

La mayoría es idiopático (80%).

Cuadro clínico variable, aproximadamente 2/3 de los pacientes mejorarán sin tratamiento.

La sordera NS súbita bilateral es muy rara y debe ser referida al especialista.

Son factores de mal pronóstico: hipoacusia intensa con pérdida en agudos, presencia de
vértigo, bilateralidad y edad avanzada.

Caso clínico tipo


Paciente hombre de 49 años, con antecedentes de DM y de haber sufrido un resfriado hace
una semana, consulta por pérdida de la audición del oído derecho, que se instaló en un
plazo de horas. Además refiere escuchar un zumbido. Examen físico normal.

Definición

Caída de la audición de tipo sensorioneural, mayor de 30 dB, que afecta a por lo menos tres
frecuencias contiguas y que se desarrolla en un período menor a tres días.

Etiología-epidemiología-fisiopatología

Se han descrito numerosas etiologías para esta enfermedad, infecciosas (viral, bacteriana,
parasitaria, micótica), vascular (Talasemia, diabetes, cirugías prolongadas, embolias,
trauma arterial, Enfermedad de Bürger, de Waldenström, vasculitis autoinmunes,
policondritis recidivante, aneurismas), autoinmune (LES, poliarteritis nodosa y el síndrome
de Cogan), traumática (fístula perilinfática, fractura del hueso temporal) y miscelánea
(Esclerosis múltiple, neurinoma del acústico, migraña, Enfermedad de Menière,
psicogénica, secundaria a uso de drogas como anticonceptivos y piroxicam); sin embargo,
no se puede precisar la causa en más del 80% de los casos, siendo considerada, en la
mayoría de ellos como idiopática. Se estima que la incidencia es de 1 cada 5000 personas al
año. La incidencia es igual en hombres y mujeres y la prevalencia es mayor entre los 40 y
50 años, aunque puede ocurrir a cualquier edad. En más del 90% de los casos, el problema
es unilateral, siendo excepcional la presentación bilateral. La sordera súbita implica un
compromiso neurosensorial, que puede originarse en el oído interno, en el octavo par
craneal o en las vías auditivas. Respecto al mecanismo, las teorías más aceptadas en la
actualidad son la infecciosa y la vascular. Infecciosa (Viral): el mecanismo de daño se
produciría por una laberintitis endolinfática viral. Merecen especial atención los virus de
parotiditis, sarampión, adenovirus, herpes zoster y gripe. Vascular: espasmo vascular,
trombosis, empastamiento, embolía, hemorragia en el oído interno e hipercoagulabilidad.
Existen además factores predisponentes, tales como: alergia, exposición a cambios físicos
ambientales (frío, cambios de presión atmosférica), alcoholismo, diabetes, arterioesclerosis,
edad avanzada y embarazo.

Diagnóstico

Si una persona ha tenido un episodio agudo de pérdida de audición, que en la evaluación


indica hipoacusia sensorioneural, sin alguna condición subyacente detectada en la historia o
el examen físico se considera que tiene sordera súbita.

Anamnesis:

Circunstancia exacta y características de la pérdida auditiva.

Antecedentes de trauma, comorbilidades y uso de fármacos.

Síntomas:
Tinnitus: en un 70 % de los pacientes, el cual puede preceder a la pérdida auditiva y llegar a
ser persistente cuando la secuela es importante.

Vértigo (cuando hay compromiso vestibular), se presenta en forma leve en un 40% de los
pacientes y en un 10% es incapacitante, suele durar 4-7 días y empeora el pronóstico de la
secuela auditiva.

Náuseas y vómitos: pueden acompañar a los síntomas anteriores.

Examen físico: en la mayoría de los casos es normal.

Exámenes audiológicos: nos darán una idea de la magnitud y evolución frente al


tratamiento efectuado.

Audiometría: confirma la hipoacusia neurosensorial

Otoscopia: descarta obstrucción del conducto auditivo externo, lesiones timpánicas y


anormalidades del oído medio.

Pruebas de Weber y Rinne: las cuales se presentarían como: Weber, que lateraliza el mejor
oído, y Rinne bilateral, hallazgos característicos de la hipoacusia neurosensorial.

Imágenes: se recomienda realizar una RM con gadolinio para descartar patología


estructural (solicitar ante una sospecha de tumor de ángulo pontocerebeloso o procesos
inflamatorios de la cóclea que justifiquen el uso de un tratamiento).

Tratamiento

Manejo:

Se debe:

Evitar la manipulación de los oídos.

Evitar traumas acústicos.

Suspender cualquier medicamento ototóxico.

Evitar la automedicación y los tratamientos caseros.

Tratamiento inicial: Cuando la causa ha sido identificada el tratamiento es específico. Sin


embargo la mayoría es idiopático, en este caso se recomienda:

Tratamiento Farmacológico.
 Si el diagnóstico ha sido posible antes de 30 días del inicio de los síntomas, el
tratamiento será mediante corticoides orales durante 1 mes, siendo tres las opciones
más habituales
o Prednisona,1mg/kg peso/día, en pauta descendente cada 5 días (por ejemplo,
en un paciente de 80kg de peso, 80mg x 5 días, 60mg x 5 días, 40mg x 5
días, 20mg x 5 días, 10mg x 5 días, 5mg x 5 días).
o Metilprednisolona, 1mg/kg peso/día, en pauta descendente cada 5 días (por
ejemplo, en un paciente de 80kg de peso, 80mg x 5 días, 60mg x 5 días,
40mg x 5 días, 20mg x 5 días, 10mg x 5 días, 5mg x 5 días).
o Deflazacort, en pauta descendente similara, 1,5mg/kg peso/día, en pauta
descendente cada 5 días (por ejemplo, en un paciente de 80kg de peso,
120mg x 5 días, 90mg x 5 días, 60mg x 5 días, 30mg x 5 días, 15mg x 5
días).
 Si hay contraindicación para el uso de corticoides sistémicos, o si no hay respuesta
al tratamiento con corticoides sistémicos, orales o intravenosos, pasados 7 días
desde su instauración”, se ofrecerá tratamiento intratimpánico con esteroides de
rescate, mediante 1 dosis semanal durante 3 semanas, en consultas externas de ORL
(y se mantendrá la pauta descendente oral), siendo dos las pautas más utilizadas
(previa anestesia tópica con fenol sobre la membrana timpánica, con aguja abocath
n° 22):
o Metilprednisolona, 0,9 cc de un vial de 40mg, mezclado con lidocaína al
1%, 0,1ml.
o Dexametasona, 0,9 cc de un vial de 8mg.
 En casos de sospecha de etiología vascular (factores de riesgo cardiovascular
conocidos, etc.), se podrán asociar vasodilatadores, como nimodipino, por vía
intravenosa (5-15 cc en 500ml de suero salino, a pasar lentamente, cada 8 horas), o
como trimetazidina, por vía oral (especialmente si el paciente ya está en tratamiento
antihipertensivo, 1 comp/8h durante 1 mes), y se remitirá el paciente al Servicio de
Medicina Interna, para su valoración y posible tratamiento antiagregante.

Si la clínica lo sugiere, también se pueden usar antivirales (aciclovir, valaciclovir), aunque


la hipoacusia súbita se recupera espontáneamente en 65% de los casos total o parcialmente,
especialmente si es menor de 40 dBs y no hay compromiso vestibular. Tratamiento
quirúrgico: en los casos en que no hay recuperación de la audición, se recomienda el
implante coclear.

Seguimiento

Derivar.

Tinnitus
Nivel de manejo por médico general: Diagnóstico Inicial.Tratamiento Inicial.
Seguimiento Derivar.
Aspectos esenciales

 Importante impacto en calidad de vida.


 Causa más frecuente: hipoacusia.
 Derivación a especialista para descartar patología orgánica subyacente.

Caso clínico tipo

Paciente varón, 60 años, con antecedentes de Hipertensión Arterial. Hace 6 meses que
percibe un sonido agudo tipo vibración, persistente, de intensidad moderada, en el oído
izquierdo, pulsátil. Refiere que le dificulta conciliar el sueño porque “no puede dejar de
escuchar el ruido”.

Definición

Percepción conciente de un sonido en ausencia de una fuente externa. Puede ser en uno o
ambos oídos, dentro o alrededor de la cabeza, o como ruido lejano. Lo más común es que
sea un zumbido, silbido o murmullo, ya sea en forma intermitente o permanente, y ser
pulsátil o no.

Etiología-epidemiología-fisiopatología

Es una patología frecuente, afectando entre un 10-15% de la población general. La


prevalencia es mayor en los hombres y aumenta con la edad.

Las teorías indican que la generación sería por una activación anormal de neuronas de la
corteza auditiva, debido a la pérdida de input coclear (por ejemplo en el daño coclear por
trauma acústico, ototoxicidad, etc).

También las estructuras cercanas a la cóclea pueden generar sonidos, y lo más frecuente es
que sean del tipo pulsátil (a menudo son estructuras vasculares, pero también
musculoesqueléticas).

Las etiologías son múltiples, abarcando desde trastornos vasculares (FAV, soplos arteriales,
paragangliomas, etc), trastornos neurológicos (disfunción tubárica, espasmos de músculos
del oído medio, disfunción de la ATM), origen en el sistema auditivo (ototóxicos,
presbiacusia, otoesclerosis, schwanoma vestibular, etc), entre otros. La condición asociada
con más frecuencia es la hipoacusia.

Diagnóstico

La anamnesis debe incluir las características del tinnitus, factores desencadenantes y


atenuantes, antecedentes de enfermedades al oído, nivel de audición exposición a trauma
acústico y/o craneal, uso de fármacos, síntomas acompañantes, entre otros.

Examen físico debe enfocarse en cabeza y cuello (incluir auscultación).


Exámenes: batería auditiva inicial (audiometría, impedanciometría, reflejo auditivo,
emisiones otoacústicas). Exámenes más específicos para descartar patología orgánica
subyacente deben ser evaluados por especialista.

Tratamiento

 Identificar y tratar causa subyacente (realizado por especialista).


 Corrección de comorbilidades (depresión, insomnio, presbiacusia, ototóxicos, etc).
 Terapias conductuales: reentrenamiento, masking.
 Fármacos: uso de Benzodiacepinas, antidepresivos, y fármacos de administración
transtimpánica: lidocaína, dexametasona, aminoglucósidos.

Seguimiento

Derivar.

Trastornos de deglución
Nivel de manejo por médico general:
Diagnóstico Sospecha.Tratamiento Inicial. Seguimiento Por especialista.

Aspectos esenciales

 Diferenciar tipo de disfagia.


 Múltiples causas.
 Anamnesis y examen físico completos.
 Derivación a especialista en forma oportuna.
 Tratar causa subyacente en lo posible.
 Reentrenamiento y rehabilitación por fonoaudiólogo.

Caso clínico tipo

Paciente 68 años, con AVE reciente, secuelado (hemiparesia FBC izquierda en


rehabilitación). Refiere que le cuesta tragar los alimentos, incluyendo la saliva, y tiene
sensación de atoro posterior a la deglución y tos. Ha notado que ha bajado de peso en el
último tiempo.

Definición

También llamada Disfagia Orofaríngea. Abarca aquellas enfermedades que afectan la


faringe y esófago superior, incluyendo el esfínter esofágico superior (EES), que como
consecuencia generan dificultad en el paso del bolo alimenticio al deglutir. Sus causas son
variadas y la gama de síntomas con que se manifiesta es muy amplia, por lo que la
evaluación de los pacientes con disfagia es compleja.
Etiología-epidemiología-fisiopatología

La incidencia está en aumento en Chile y el mundo, entre otras causas por el aumento de la
población mayor de 65, la mayor presencia de comorbilidades asociadas, y la mejor
sobrevida de patologías que provocan secuelas, como el accidente vascular encefálico
(AVE), que pueden presentar disfagia hasta en 80% de los casos, dependiendo del método
diagnóstico utilizado. Entre los diagnósticos diferenciales se encuentran:

 Neoplasias: tumores del tracto aero-digestivo superior, cáncer de tiroides, tumores


del Sistema Nervioso Central
 Neurológica: AVE, esclerosis múltiple, enfermedad de parkinson, esclerosis lateral
amiotrófica, miopatías.
 Sistémicas: dermatomiositis, esclerodermia, amiloidosis, sarcoidosis.
 Congénitas: fístula traqueo-esofágica, hendidura laríngea.
 Traumáticas: SNC, cuello.
 Inflamatorias: RGE.
 Infecciosas: enfermedad de Chagas/acalasia, enfermedad de Lyme.
 Endocrinológicas: bocio, hipotiroidismo, neuropatía diabética.
 Iatrogénicas: cirugías cardiacas, cirugías de cuello y esófago, cirugías de base de
cráneo, quimioterapia, radioterapia en cabeza y cuello, medicamentos.
 Presencia de traqueostomía.
 Cirugías de resección de cánceres de cavidad oral/orofaringe/hipofaringe,
divertículo de Zenker, etc.

La alteración puede estar en dos niveles:

Fase oral preparatoria: mala dentadura, flujo salival disminuido, mucosa orofaríngea
alterada, y trastornos neurológicos (como AVE o Enf de Parkinson).

Fase faríngea: descoordinación neuromuscular (2ario a alteraciones en SNC como AVE o


ELA, o por alteraciones en SNP como Miastenia Gravis o parálisis de Bell), obstrucción a
nivel orofaríngeo (tumores principalmente, ya sean benignos o malignos) y trastornos de
relajación del EES.

Diagnóstico

Síntomas habituales tienen relación con la dificultad para deglutir tanto sólidos como
líquidos, que incluso pueden llegar hasta el ahogo y la aspiración, inmediatamente después
de la deglución. La evolución puede ser progresiva, repercutiendo a nivel sistémico (baja de
peso y deshidratación). Indagar sobre síntomas asociados: fiebre, baja de peso, dolor al
tragar, pirosis, masas cervicales, sialorrea, regurgitación nasal, tos, carraspera, disnea,
sequedad bucal u ocular, cambios en la voz y habla y síntomas en la esfera neurológica que
nos hagan pensar en alguna patología específica. También es importante establecer
antecedentes de abuso de OH y tabaquismo, radioterapia de cabeza y cuello, intubación
reciente, entre otros.
Examen físico: examen neurológico (que considere marcha, sensibilidad/fuerza de
extremidades, reflejos tendíneos y nervios craneanos), examen de boca y orofaringe que
evalúe movilidad y sensibilidad lingual y del velo del paladar, continencia oral, manejo de
la saliva, masas, voz y habla y de cuello.

Exámenes específicos (radiografía con bario, descartar Espasmo Esofágico Difuso,


laringoscopía por fibra óptica, endoscopía, manometría esofágica, videofluoroscopía)
pueden ser resorte de especialistas.

Tratamiento

Corregir la patología de base, con oportuna derivación a especialista. Medidas generales


(reducción de volumen y tipo de comida, alimentación asistida, etc). Además es útil el
reentrenamiento y rehabilitación del acto deglutorio por parte de fonoaudiólogo.

Seguimiento

Por especialista, acorde a la patología de base.

Trastornos del habla y del lenguaje


Nivel de manejo del mèdico general: Diagnòstico Sospecha Tratamiento Inicial
Seguimiento Por especialista

Aspectos esenciales

 Patología frecuente en la infancia.


 Consecuencias importantes en distintas áreas del desarrollo.
 Descarte y tratamiento de condiciones médicas subyacentes.
 Siempre descartar déficit auditivo.
 Enfoque multidisciplinario.

Caso clínico tipo

Niño de 3 años, con DSM normal, pero que en el área del lenguaje los padres refieren que
siempre ha costado que personas extrañas le entiendan, aunque ellos manifiestan no tener
grandes dificultades al respecto, definiéndolo como “regalón”.

Definición

Corresponde al deterioro en la capacidad de recibir, enviar, procesar y comprender


conceptos o sistemas de símbolos gráficos, verbales y no verbales. Los trastornos del habla
abarcan la alteración de la articulación de los sonidos, la fluidez y/o la voz. Los trastornos
del lenguaje se refiere a la alteración en la comprensión y/o uso del lenguaje hablado,
escrito y/u otros sistemas de símbolos.

Es necesario reconocerlos y tratarlos tan pronto sea posible, para dar la mayor oportunidad
de superar el impedimento antes de que comience su educación básica.

Etiología-epidemiología-fisiopatología

Relativamente frecuente en la infancia, con una prevalencia de 5-10%, siendo el trastorno


de lenguaje el más común.

La importancia radica en su asociación a un rendimiento escolar deficiente, así como


también a trastornos conductuales y emocionales.

Cuando un profesional se encuentra frente a un niño sin un desarrollo normal del lenguaje,
lo primero es identificar si el deterioro del lenguaje es primario, o secundario a un problema
generalizado. Por lo que en el preescolar se debe plantear como diagnósticos diferenciales:

1. Déficit auditivo
2. Déficit cognitivo
3. Disfasia
4. Trastorno del espectro autista.

Trastornos del habla: pueden ser por compromiso de la articulación (debido a discapacidad
auditiva, problemas neurológicos, apraxia, defectos anatómicos), de la fluidez o de la voz
(incluyendo defectos en la resonancia).

Trastorno del lenguaje: existen dos subtipos: expresivo, y expresivo receptivo (mixto). Se
clasifican en trastorno de desarrollo del lenguaje y trastorno específico del lenguaje, el que
poseería un probable componente genético. Las etiologías adquiridas abarcan infecciones
(OME), trastornos neurológicos degenerativos, trauma craneal, negligencia y abuso.

Diagnóstico

Lo más importante es una búsqueda activa, poner especial atención a los hitos del DSM del
niño, con derivación inmediata en caso de aparición de signos de alarma.

Debe obtenerse los antecedentes médicos del niño y de ambos padres, informe escolar y en
lo posible observación del niño en su entorno habitual.

Realizar examen físico completo, incluyendo exámenes para detectar condiciones médicas
que contribuyan a la discapacidad del habla (ej: audiometría, impedanciometría o
potenciales evocados).

Tratamiento
Tratar patologías médicas asociadas. Realizar intervenciones específicas, en conjunto con
especialistas, como por ejemplo: inscripción en terapia de habla y lenguaje, ya sea
individual o en grupo; asistencia a escuelas especializadas para niños con trastornos del
habla y lenguaje; evaluaciones adicionales en áreas específicas (por ejemplo, oral, motor,
psicológico).

Seguimiento

Por especialista.

Vous aimerez peut-être aussi